Sie sind auf Seite 1von 38

A Sample Questions

A Collective Arrangement by the Chapter Editors, Associate Editors,


and CD-ROM Editor of 2005

COMMUNITY AND PUBLIC HEALTH . . . . . . . . . . . . . . . . . . . .2

MEDICINE . . . . . . . . . . . . . . . . . . . . . . . . . . . . . . . . . . . . . . . . . . . .6

OBSTETRICS AND GYNECOLOGY . . . . . . . . . . . . . . . . . . . . . .11

PEDIATRICS . . . . . . . . . . . . . . . . . . . . . . . . . . . . . . . . . . . . . . . . . .14

PSYCHIATRY . . . . . . . . . . . . . . . . . . . . . . . . . . . . . . . . . . . . . . . . .20

SURGERY . . . . . . . . . . . . . . . . . . . . . . . . . . . . . . . . . . . . . . . . . . . .24

ANSWERS TO SAMPLE QUESTIONS . . . . . . . . . . . . . . . . . . .29

KEY FEATURE QUESTIONS . . . . . . . . . . . . . . . . . . . . . . . . . . . .31

KEY FEATURE SCORING GUIDE . . . . . . . . . . . . . . . . . . . . . . .36

Toronto Notes 2005 Sample Questions 1


2 Sample Questions Community and Public Health Toronto Notes 2005

Community and Public Health


1) Which one of the following is not a typical feature of asbestosis? 9) In a cohort study of disease “X” in people with risk factor
a) increased risk of cancer “Y” versus those who are without risk factor “Y”, the
b) pleural thickening and calcification following results were obtained:
c) interstitial fibrosis
d) obstructive pattern on pulmonary function tests X no X total
e) none of the above
Y 80 20 100
2) The following statements regarding contact dermatitis are true no Y 50 50 100
EXCEPT:
a) phototoxic dermatitis following topical application of The relative risk of developing “X” in “Y” versus no “Y” is:
creosote requires UV light
b) photoallergic contact dermatitis requires UV light to be a) 80 x 50 = 4 c) 50 x 80 = 4
manifested 50 x 20 20 x 50
c) contact eczema involves a type IV delayed
hypersensitivity reaction b) 50 _ 20 = 0.3 d) 80/100 = 1.6
d) contact urticaria or hives is a common form of dermatitis 100 100 50/100
e) chemical burns by HCl and KOH may result in an irritant
contact dermatitis e) it is not possible to calculate risk in a cohort study

3) The following statements regarding noise are true EXCEPT: 10) The attributable risk of factor “Y” to disease “X” would be:
a) temporary threshold shift recovers following cessation of
noise exposure a) 80/100 = 1.6 c) 80 x 50 = 4
b) permanent threshold shift is characterized by a 50/100 50 x 20
progressive pattern of hearing loss
c) most cases of permanent threshold shift are surgically treatable b) 80 _ 50 = 0.3 d) 50 x 20 = 0.25
d) higher frequency noise is more damaging than low frequency 100 100 80 x 50
noise
e) none of the above e) it is not possible to calculate atrributable risk
in a cohort study
4) The frequencies most necessary for the understanding of
speech extend from about: 11) A group of 50 people are exposed to virus “A”. Of those 50
a) 20-20 000 Hz people, 9 develop a mild infection, 10 become seriously ill, and 3
b) 400-4 000 Hz die. The attack rate of virus “A” in the population would be:
c) 250-8 000 Hz a) 22/50
d) 100-5 000 Hz b) 9/50
e) none of the above c) 10/50
d) 19/50
5) Lead exposure typically results in: e) 13/50
a) chronic dermatitis
b) resting and intention tremor 12) The following indicate the results of screening test “Q” in
c) extensor muscle weakness screening for disease “Z”:
d) arrhythmias Disease Z
e) cerebellar ataxia + –
6) Which of the following statements concerning the Worker’s + 40 10 50
Compensation Act is true? Screen Q – 30 120 150
a) the worker reserves the right to sue the employer for negligence
b) funding is provided by the provincial government 70 130 200
c) the worker is guaranteed payment from the first day of
injury/illness if it is deemed to be work-related The specificity of test “Q” would be:
d) the Worker’s Compensation Board is an independent, private a) 40/70
agency b) 120/130
e) none of the above c) 40/50
d) 120/150
7) Which of following statements regarding radiation is false? e) 40/130
a) natural background radiation accounts for about half of a
typical person’s exposure 13) The positive predictive value would be:
b) ionizing radiation causes intestinal villi to become denuded a) 40/70
c) exposure to non-ionizing radiation may result in cataracts b) 120/130
d) ionizing radiation results in an increased incidence of c) 40/50
neoplasia such as lung and thyroid d) 120/150
e) none of the above e) 70/200
8) Which statement concerning vibration induced white 14) To determine an odds ratio one would have to perform
finger disease is false? which of the following studies?
a) early symptoms include tingling and numbness of the a) a cross sectional/prevalence study
fingers when at rest b) a randomized controlled trial
b) swelling of the fingers over knuckles may be an early c) a cohort study
feature d) a case study
c) cold, damp conditions may precipitate symptoms e) a case control study
d) the affected area eventually spreads to involve all fingers
e) none of the above
Toronto Notes 2005 Community and Public Health Sample Questions 3

15) Examples of secondary prevention would include all of the 23) Which of the following is the most important justification for
following EXCEPT: population screening programs for a specific disease?
a) Pap smear for cervical cancer a) early detection of the disease of interest is achieved
b) chemoprophylaxis in a recent TB converter b) the specificity of the screening test is high
c) proctoscopy for rectal cancer c) the natural history of the disease is favourably altered by
d) immunization for Haemophilus influenzae B early detection
e) mammography for breast cancer d) effective treatment is available
e) the screening technology is available
16) Alpha error is:
a) the probability of declaring a difference to be absent 24) Regarding the regulation of health professionals, provincial
when it in fact is present colleges of physicians and surgeons:
b) the probability of declaring a difference to be present a) have the advancement of the public interest as their
when it is not primary goal
c) the probability of declaring a difference to be absent b) protect the public from incompetent or unfit MDs
when it is indeed absent c) act as licensing bodies for MDs
d) the probability of declaring a difference to be present d) do not advance the professional and political interests of MDs
when it does exist e) all of the above

17) Which one of the following descriptors of a diagnostic test is 25) Active immunization was important in control of each of the
influenced by the prevalence of the disease being tested for: following childhood communicable diseases EXCEPT:
a) specificity a) diphtheria
b) sensitivity b) polio
c) accuracy c) measles
d) positive predictive value d) scarlet fever
e) reliability e) pertussis

18) Which of the following statements regarding the measurement 26) All of the following statements are true EXCEPT:
of health and disease in a population is true? a) one indirect measure of a population’s health status is the
a) a rate is the number of times an event has occured percentage of low birth weight neonates
during a certain time interval divided by the number of b) accidents are the largest cause of potential years of life lost
persons at risk during the same interval in Canada
b) when calculating a ratio, the numerator is a portion of c) the Canadian population is steadily undergoing
the denominator rectangularization of mortality
c) a ratio is the number of times an event has occurred d) morbidity is defined as all health outcomes excluding death
during a certain time interval multiplied by the number e) the neonatal mortality rate is the number of infant deaths
of times an event has occurred during the same interval divided by the number of live births multiplied by 1000
d) when calculating a rate, the numerator is not a portion of
the denominator 27) All of the following statements are true EXCEPT:
e) none of the above a) the data collected on a death certificate is uniform and in
conformity with WHO guidelines
19) In 1990, which country spent the least on health care as a b) Section 7 of the Coroner’s Act states that the coroner’s office
percentage of GDP (gross domestic product): must be notified if a patient dies after some mishap such
a) Canada as leaving an instrument in the body at surgery
b) France c) diseases which must be reported to the local medical
c) Sweden officer of health include AIDS, food poisoning, influenza
d) UK and gonorrhea
e) USA d) a tuberculin reaction greater than 5 mm is considered
positive in all individuals
20) The component of Canada's health care system that e) all of the above
receives the highest percentage of the health care budget is:
a) hospitals 28) In describing the leading causes of death in Canada, two very
b) physician fees different lists emerge, depending on whether proportional
c) drug benefit plans mortality rates or person-years of life lost (PYLL) are used. This
d) laboratory services is because:
e) administration a) one measure uses a calendar year and the other a fiscal
year to calculate annual experience
21) The British North America Act (BNA): b) one measure includes morbidity as well as mortality
a) was mainly concerned with health care legislation experience
b) granted exclusive powers over health care to the c) both rates exclude deaths occurring over the age of 70
provinces, including powers over marine hospitals d) different definitions of “cause of death” are used
c) replaced the Canada Health Act e) one measure gives greater weight to deaths occurring in
d) applied to British Columbia only younger age groups
e) none of the above
29) Differentiation between a point-source epidemic and a
22) Regarding health expenditure and health outcomes: progressive (propagated) epidemic is made by:
a) the U.S. has the greatest health expenditure and the a) considering the characteristics of the infectious agent
lowest infant mortality rates b) determining the level of immunity in the community
b) there is a positive association between national c) determining the number of persons infected and
expenditure on health and GDP calculating the attack rate
c) increased national health expenditure always increases d) plotting the distribution of cases by time onset
health status of a country e) none of the above
d) all of the above
e) none of the above
4 Sample Questions Community and Public Health Toronto Notes 2005

30) The occurrence of an illness at a rate of above that expected is 38) In 1981, the crude birth rate in Ontario was approximately
called: 14 per 1000 and the crude death rate was 7 per 1000. The
a) hyperendemic estimated rate of net migration was –1 per 1000. The growth
b) epidemic rate of the province, per 1000 population was:
c) endemic a) 6
d) enzootic b) 7
e) pandemic c) 8
d) 20
31) Each of the following statements applies to case control e) 22
studies EXCEPT:
a) starts with disease 39) All of the following statements about environmental
b) suitable for rare diseases health are true EXCEPT:
c) relatively inexpensive a) levels of toxic agents measured in the environment may not
d) prolonged follow-up required reflect internal organ levels
e) there may be a problem in selecting and matching controls b) the federal government monitors the quality and types of
industrial emissions and toxic waste disposal
32) A clinician who has been examining the patterns of mortality in c) sick building syndrome is associated with Pontiac fever and
your community says that the rates for heart disease and lung Legionnaire’s disease
cancer are higher in this community than in an adjacent d) all humans have detectable levels of PCBs
community. Which of the following questions should you ask first? e) none of the above is true
a) how did the clinician choose the comparison community?
b) have the rates been standardized for age? 40) The effectiveness of a preventative measure is assessed in terms of:
c) are tobacco sales significantly different in the two communities? a) the effect in people to whom the measure is offered
d) are the facilities to treat these diseases comparable in the b) the effect in people who comply with the measure
two areas? c) availability with the optimal use of resources
e) are the numbers of deaths comparable in each area? d) the cost in dollars versus the benefits in improved health status
e) all of the above
33) The purpose of randomization is to:
a) make sure that there are equal numbers of men and women 41) All of the following statements about the Canada Health
in test and control groups Act (1984) are true EXCEPT:
b) increase the chances of getting a statistically significant difference a) it did not define all medically necessary hospital and
c) ensure that the numbers of cases and controls are equal physician services
d) limit bias b) the CHA replaced the Hospital Insurance and Diagnostic
e) all of the above Services Act of 1957
c) the CHA banned all forms of extra billing
34) Which of the following types of studies usually provides only a d) according to the CHA, provinces must meet all the terms
measure of prevalence? and conditions of Medicare to qualify for federal transfer
a) descriptive payments
b) cross-sectional e) none of the above
c) randomized controlled trial
d) cohort 42) Each of the following is an example of primary prevention EXCEPT:
e) none of the above a) genetic counselling of parents with one retarded child
b) nutritional supplements in pregnancy
35) The major advantage of cohort studies over case-control c) immunization against tetanus
studies is that: d) chemoprophylaxis in a recent tuberculin converter
a) they take less time and are less costly e) speed limits on highways
b) they can utilize a more representative population
c) it is easier to obtain controls who are not exposed to the factor 43) The classical “epidemiological triad” of disease causation consists of
d) they permit estimation of risk of disease in those exposed factors which fall into which of the following categories:
to the factor a) host, reservoir, environment
e) they can be done on a “double-blind” basis b) host, vector, environment
c) host, agent, environment
36) All of the following statements concerning occupational d) reservoir, agent, vector
health are true EXCEPT: e) host, age, environment
a) disorders of reproduction are among the top 10
work-related diseases and injuries 44) Of the five items listed below, the one which provides the strongest
b) most workers are covered by both federal and provincial evidence for causality in an observed association between exposure
legislation with respect to workplace health and safety and disease is:
c) skin problems and hearing problems together are a) a large attributable risk
responsible for half of WCB claims b) a large relative risk
d) a complete occupational medical history includes c) a small p-value
investigation of the temporal relationship between d) a positive result from a cohort study
symptoms and exposure e) a case report

37) Which of the following statements concerning exposure to solvents 45) The difference between a common point source outbreak and
in the workplace is true? a propagated outbreak of illness is that:
a) each solvent compound has a specific antidote that can be a) all cases in a common point source outbreak occur within
used to treat exposure one incubation period of the exposure
b) a prominent symptom of solvent exposure is memory loss b) the attack rates in propagated outbreaks are higher
c) some solvents can cause skin dryness and loss of c) person-to-person transmission is a feature of common
subcutaneous adipose tissue source outbreaks
d) solvents do not affect the bone marrow d) case fatality rates in common source outbreaks are generally
e) all of the above higher
e) the source of infection in propagated outbreaks is more
easily contained than in common source outbreaks
Toronto Notes 2005 Community and Public Health Sample Questions 5

46) Which of the following are strategies for control of disease: 50) All of the following are responsibilities of local public health
a) population immunization units in Canada EXCEPT:
b) contact tracing to offer treatment to all who could be infected a) communicable disease control
c) monitoring increases in the population of certain disease vectors b) health education
d) having physicians report curable, potentially serious diseases c) investigation of sudden death
e) all of the above are true d) immunization
e) health promotion
47) During a clinical trial, the difference in the success rates of two
drugs was not statistically significant. This means that: 51) Who is ultimately responsible for the quality of care in a hospital?
a) there is no difference in drug effectiveness a) chief executive officer
b) there is a sizable probability that the demonstrated difference in b) board of trustees
the drugs’ effectiveness could occur due to chance alone c) medical director
c) the demonstrated difference in the drugs’ effectiveness is d) chief of staff
too small to be clinically meaningful e) attending physicians
d) the two samples of patients on which the drugs were tested
came from the same population 52) Which of the following is not one of the 5 Terms and
e) none of the above are true Conditions of Medicare?
a) portability
48) The incidence of a particular disease is greater in men than in b) flexibility
women, but the prevalence shows no sex difference. The most c) universality
probable explanation is that: d) comprehensive coverage
a) the mortality rate is greater in women e) accessibility
b) the case fatality rate is higher in women
c) the duration of the disease is longer in women 53) You are given this data pertaining to an outbreak of diarrhea
d) women receive less adequate medical care for the disease in a daycare:
e) this diagnosis is more often missed in women Age Number of Children Number of Diarrhea
1 20 17
49) All of the following statements about statistical tests are true 2 19 15
EXCEPT: 3 39 13
a) linear regression is used to describe the relationship 4 39 4
between two continuous variables 5 38 5
b) a confidence interval is a range of values giving information 6+ 18 1
about the precision of a measurement
c) ANOVA tables are used to make comparisons among the means What is the attack rate of this illness?
of 3 or more groups simultaneously a) 25% d) 40%
d) in a normal distribution, the mean, median and mode are equal b) 30% e) 50%
e) the chi-square test evaluates the statistical significance of 2 c) 32%
or more percentages of categorical outcomes
6 Sample Questions Medicine Toronto Notes 2005

Medicine
1) Which of the following is a feature of secretory diarrhea? 9) Hemolytic anemia is characterized by all of the following EXCEPT:
a) small stool volume (< 1L/day) a) increased LDH
b) increased stool osmotic gap b) increased reticulocytosis
c) persistent diarrhea despite fasting c) increased unconjugated bilirubin
d) blood and/or pus in stools d) increased haptoglobin
e) malodorous, often floating stools e) lead poisoning

2) Each of the following are risk factors for colon cancer EXCEPT: 10) A patient complains of a non-tender mass over the thyroid region
a) low fiber diet on the left side of her neck. Concerned about a thyroid disorder,
b) severe diverticular disease you order the appropriate investigations. The results are as
c) familial adenomatous polyposis follows:
d) ulcerative colitis TSH: 6.0
e) high fat diet Free T4: 20.2
Thyroid antibodies: none
3) Intestinal complications more common to Crohn’s disease than RAIU: No “hot” spots seen
ulcerative colitis include each of the following EXCEPT:
a) fistula formation The next investigation(s) you choose to do are:
b) perianal disease a) watch and wait for 3-6 months
c) intestinal obstruction b) FNA
d) toxic megacolon c) surgical biopsy
e) post-surgical recurrence d) trial of L-thyroxine therapy for 6 months
e) none of the above
4) A 21 year-old bisexual man has a 4 week history of intermittent
diarrhea, urethral discharge, and pain in the right knee and left 11) An 8 year-old boy is brought to the office because his mother is
second toe. He has several oral ulcers, a clear urethral discharge, a concerned he is entering puberty already. You examine him and
scaly papular rash on palms and soles, onycholysis, sausage-like note the beginnings of facial hair, axillary hair and Tanner stage 2
swelling of the left second toe, and heat and swelling of the right external genitalia. Choose the set of investigations you initially
knee. The results of Gram stains and cultures of urethral discharge want to do:
are negative. Rheumatoid factor is not present. The most likely a) CBC, lytes, testosterone, bone age, CT head
diagnosis is: b) FSH, LH, testosterone, lytes, bone age, DHEA-S
a) Reiter’s syndrome c) FSH, LH, testosterone, cortisol, DHEA-S, 11-OH progesterone,
b) gonococcal arthritis bone age
c) Behcet disease d) lytes, testosterone, DHEA-S, 17-OH progesterone, cortisol, bone
d) acquired immune deficiency syndrome age
e) psoriatic arthritis
12) The subendothelium is the most vulnerable segment of the heart
5) A 53 year-old presents to your office with pain and stiffness in from an ischemic standpoint. The major reason for this is:
both hands and knees of 6 months duration. Which of the a) the highest oxygen utilization is in the subendocardium
following findings on your physical examination may help with a b) coronary flow to the subendocardium occurs almost completely
diagnosis: during diastole whereas other regions receive some flow
a) joint tenderness/effusions during systole as well
b) maculopapular rash c) the subendocardium has a diminished aerobic capacity
c) iridocyclitis d) there is less potential for collateralization to the
d) hepatosplenomegaly subendocardium
e) all of the above e) the ratio of capillary to myocyte is less in the subendocardium
than in other regions
6) Radiographic features of osteoarthritis of the knee include
which of the following: 13) 18 year-old female with initial onset of pruritic rash characterized
a) marginal erosions by excoriations, scaling and crusting and distributed on the
b) juxta-articular osteopenia (demineralization) extremities, neck and eyelids. Past medical history is significant for
c) loss of articular cartilage with narrowing of the radiologic asthma and hayfever. The most likely diagnosis is:
joint space a) scabies
d) osteonecrosis (avascular necrosis) of the medial femoral b) atopic dermatitis
condyle c) contact dermatitis
e) high riding patella (patella alta) d) shingles
e) dyshydrotic eczema
7) An elevated level of hemoglobin A2 in a patient with mild
microcytic anemia suggests the diagnosis of: 14) 8 year-old black male comes in with an asymptomatic
a) alpha-thalassemia erythematous eruption characterized by oval patches with collarette
b) sickle trait scaling. It is distributed as a "Christmas tree" pattern on the back.
c) beta-thalassemia Father states that there was originally one lesion on the abdomen a
d) hereditary spherocytosis few weeks prior. What is the most likely diagnosis:
e) hereditary persistence of fetal hemoglobin a) pityriasis rosea
b) tinea corporis
8) It is unlikely to see macrocytosis in a patient with anemia in which c) lichen planus
of the following? d) psoriasis
a) reticulocytosis e) none of the above
b) vitamin B12 deficiency
c) folate deficiency
d) myelodysplastic syndrome
e) sideroblastic anemia
Toronto Notes 2005 Medicine Sample Questions 7

15) 30 year-old female comes in with a soft smooth erythematous 24) Which of the following is not an aggravating factor of congestive
nodule on her lower lip. She states that a few weeks prior she had heart failure?
some chapped lips with occasional bleeding. Now, the lips have a) hypertension
healed but this lesion arose suddenly in its place. It is occasionally b) thyrotoxicosis
tender on pressure. The most likely diagnosis is: c) alcohol
a) HSV1 d) inactivity
b) cherry hemangioma e) arrhythmia
c) pyogenic granuloma
d) dermal nevus 25) In the course of DKA, serum potassium levels:
e) none of the above a) remain unaffected
b) can appear normal but total body potassium may actually be
16) A 65 year-old male with back pain, nephrotic syndrome and low
anemiapresent to the ER. Ultrasound shows normal kidney size. c) can appear normal but total body potassium may actually be
His creatinine is 500. Which diagnosis best fits the scenario? high
a) polycystic kidney disease d) will naturally be corrected by insulin administration
b) chronic GN e) none of the above
c) multiple myeloma
d) diabetic nephropathy 26) A 2 month-old boy has a Ca of 1.80 (corrected) after an assessment
e) analgesic abuse for FTT. Mother informs you she has been consistently
breastfeeding without trouble as corroborated by a visiting nurse.
17) Which of the following are indications for dialysis in ARF? What is at the top of your differential?
a) severe alkalosis unresponsive to medical therapy. a) malabsorption
b) severe acidosis unresponsive to medical therapy. b) pseudohypoparathyroidism
c) severe hypokalemia unresponsive to medical therapy. c) mother didn’t supplement with DiVisol (Vit D supplement)
d) severe hypercalcemia unresponsive to medical therapy. d) DiGeorge syndrome
e) b and d
27) A virus that is not inactivated by mild detergents that solubilize
18) Which of the following is true with respect to diabetes and kidney phospholipid membranes is:
disease? a) poliovirus
a) primarily affects the tubules. b) variola virus
b) earliest sign is decreased GFR. c) cowpox virus
c) microalbuminuria is a late sign of DM nephropathy. d) vaccina virus
d) threshold for dialysis is same as other CRF patients.
e) BP control slows progression of DM nephropathy. 28) All the following are true statements about viruses EXCEPT:
a) they are obligate intracellular parasites
19) The various species of Campylobacter can cause diseases ranging b) they are filterable agents
from acute enteritis to bacteremia. Which of the following modes of c) they are simply organized
transmission does NOT apply to Campylobacter? d) they are devoid of enzymes
a) contact with infected animals e) they may contain double stranded DNA
b) contaminated food and water
c) improperly cooked poultry 29) A viral genome that does not replicate in the cytoplasm of the
d) aerolized droplets infected cell is:
e) person to person spread via fecal-oral route a) poliovirus
b) rabies virus
20) Which of the following is NOT a common infectious cause of acute c) cytomegalovirus
diarrhea? d) rubella virus
a) Escherichia coli e) mumps virus
b) Shigella
c) Norwalk virus 30) Which of the following is true about congenital heart block in
d) Vibrio cholerae neonatal lupus erythematosus?
e) Helicobacter pylori a) it is associated with maternal anti-Ku autoantibodies
b) it is transient
21) Impaired coronary flow reserve is associated with each of the c) the majority of patients will require a pacemaker
following conditions EXCEPT: d) there is no increased risk of connective tissue disease in
a) severe aortic stenosis adulthood
b) severe systemic hypertension with left ventricular hypertrophy e) the risk of mortality is small
c) severe mitral stenosis in the presence of atrial fibrillation
d) a totally occluded coronary artery but with excellent collateral 31) Which of the following is true about serologic testing in SLE?
supply from the contralateral (i.e. opposite) coronary artery a) Aapositive ANA is specific for SLE
e) an isolated 30% diameter stenosis of a coronary artery b) ds-DNA level correlates with disease activity in SLE
c) anti-histone antibodies are seldom positive in non-drug induced
22) Which of the following pulmonary function tests most SLE
reliably discriminates “pure” chronic bronchitis from d) the majority of patients with SLE have anti-Sm antibodies
emphysema? e) anti-Ro antibody is specific for SLE
a) total lung capacity
b) functional residual capacity 32) The treatment of choice for thrombotic events in the
c) residual volume antiphospholipid antibody syndrome is:
d) single breath diffusing capacity a) intravenous steroids
e) flow at 50% vital capacity b) high-dose oral steroids with a rapid taper
c) penicillamine
23) Which of the following would NOT be part of your plan for the d) aspirin
treatment of acute ventricular fibrillation? e) warfarin
a) electrical defibrillation
b) lidocaine
c) epinephrine
d) bretylium
e) manganese
8 Sample Questions Medicine Toronto Notes 2005

33) Highly infective chronic hepatitis B is suggested by: 41) Which of the following pair of CNS lesions and corresponding
a) elevated liver enzymes, HBeAg+, anti-HBc IgM+ visual field defects is incorrect?
b) normal liver enzymes, HBeAg+, anti-HBc IgG+ a) temporal lobe tumour – superior quadrantanopia
c) normal liver enzymes, HBeAg-, anti-HBc IgG+ b) frontal lobe tumour – altitudinal field defect
d) elevated liver enzymes, HBeAg+, anti-HBc IgG+ c) pituitary tumour – bitemporal hemianopsia
d) occipital lobe tumour – homonomous hemianopsia
34) Which is more often associated with hospital acquired pneumonia e) Multiple Sclerosis – central scotoma
than community acquired pneumonia?
a) Streptococcus pneumoniae 42) The “triple bolus” test of pituitary function works by a rapid
b) Hemophilus influenza succession of IV constituents as follows:
c) Legionella a) insulin – hypoglycemia mediated rise in GH and ACTH
d) Chlamydia pneumoniae GHRH – rise in LH and FSH
e) Mycoplasma pneumoniae TRH – rise in TSH and PRL
b) CRH – rise in GH and ACTH
35) Which of the following is least likely to contribute to myeloma? GHRH – rise in LH and FSH
a) hypercalcemia TRH – rise in TSH and PRL
b) amyloidosis c) estrogen – rise in LH, drop in FSH and PRL
c) infiltration of the kidney by myeloma cells insulin – rise in GH and ACTH
d) hyperuricemia TRH – TSH
e) intratubular light chain deposition d) cortrosyn – rise in GH and ACTH
GHRH – rise in LH and FSH
36) All of the following are vitamin-K dependent proteins EXCEPT: TRH – rise in TSH and PRL
a) protein C
b) antithrombin III 43) A 58 year-old man with a past history of a parathyroidectomy for
c) factor IX primary hyperparathyroidism is now in your office complaining
d) factor II of headaches worse in the AM (made worse by a small MVA he
e) factor VII credits to a loss of peripheral vision). You plan to:
a) send to the Emergency Department for an immediate CT head
37) An 11 year-old male comes in with erythematous pustules, b) check his calcium to ensure there’s no remaining parathyroid
inflamed nodules and cysts with some scaring distributed on the tissue
face predominantly. Diagnosis of acne vulgaris was given. c) check for a pheochromocytoma (which you know causes
Topical erythromycin was used for 2 weeks, several months ago, H/As) because you are concerned he has MEN I syndrome
with no response. What treatment would you prescribe now? d) check for a homonymous hemianopia because you are
a) accutane immediately worried about a pituitary tumor
b) topical tretinoin e) check for a bitemporal hemianopia because you are worried
c) topical benzoyl peroxide about a pituitary tumor
d) topical antibiotic other than erythromycin
e) oral antibiotic 44) A 30 year-old patient with asthma complains of daily wheezing
f) oral antibiotic and topical tretinoin and occasional waking at night with cough and chest tightness for
three weeks. His usual medication is salbutamol two puffs
38) A 40 year-old woman develops recurrent papules and pustules in tid-qid. The next step in management is:
a symmetrical pattern on her cheeks, nose, chin and forehead. She a) add long-term theophylline
blushes easily, especially when consuming hot liquids, alcohol, or b) increase salbutamol to two puffs q4h
spicy foods. The most likely diagnosis is: c) add ipratropium bromide two puffs qid
a) acne vulgaris d) add beclomethasone two puffs qid
b) perioral dermatitis e) discontinue salbutamol and begin prednisone 50 mg od and
c) acne rosacea taper over 2 weeks
d) seborrheic dermatitis
e) carcinoid syndrome 45) Which is not a feature of asbestosis?
a) Increased risk of cancer.
39) A 27 year-old man is brought into the ER after a bicycling accident. b) Pleural thickening and effusion.
A car door suddenly opened in front of him, of which he smashed c) Interstitial fibrosis.
into and was thrown 15 feet. On examination, he is drowsy and d) Obstructive pattern on pulmonary function tests.
confused. He opens his eyes when his name is called. He mumbles e) All of the above are features of asbestosis.
words that you understand but the sentences do not make sense.
He moves all four limbs but does not respond to any commands. 46) A 63 year-old woman develops intermittent dizziness.
He is able to pull both hands away when pinched and squirms Examination discloses diminished corneal light reflex and mild
when his sternum is rubbed, making no effort to stop you. What is hearing loss in theright ear. The most likely diagnosis is:
his Glasgow COMA Scale score? a) cerebellopontine angle tumour
a) 10 b) benign paroxysmal positional vertigo
b) 11 c) lateral medullary syndrome
c) 9 d) Méniére disease
d) 8 e) none of the above
e) 7
47) A 25 year-old man is admitted with a history suggesting seizures.
40) A 74 year-old, right-handed man presents with a past medical Which of the following would not support this diagnosis?
history of hypertension and dyslipidemia for 30 years. He is a a) urinary incontinence
retired banker who recently has had trouble calculating his b) the sound of voices preceding events
restaurant bill. He also notices that his writing has deteriorated. c) drowsiness and weakness following the event
On physical exam, he has difficulty naming his fingers and is d) rarely occur when recumbent
confused with distinguishing left from right. The lesion is most e) none of the above
likely in which part of the brain?
a) right parietal 48) Which of the following would not be expected in a right-sided
b) left parietal Brown-Séquard syndrome?
c) left temporal a) right-sided hemi-paresis
d) right temporal b) right-sided decreased proprioception
e) frontal c) left-sided decreased sensitivity to pinprick
d) left-sided decreased vibration sense
e) none of the above
Toronto Notes 2005 Medicine Sample Questions 9

49) Endocarditis in an I.V. drug user: 59) Which of the following is NOT an age-related change?
a) is equally prevalent to that of the normal population a) impaired myocardial diastolic dysfunction
b) is commonly located in the mitral valve b) increased gastric acid secretion
c) is typically caused by S. pneumoniae c) decreased drug clearance
d) is typically found on the tricuspid valve d) increased nocturnal sodium and fluid excretion
e) is typically found on the aortic valve, producing a systolic e) decreased baroreflex sensitivity
ejection murmur
60) Regarding the elderly patient, which of the following apply?
50) A 55 year-old man with a history compatible with chronic a) vague symptoms
bronchitis presents to your office with shortness of breath on exam. b) atypical presentations
In the history, all of the following would be anticipated EXCEPT: c) loss of function
a) A 20-year history of smoking d) polypharmacy
b) worsening of symptoms with exposure to smog e) all of the above
c) worsening of symptoms with acute respiratory infections
d) recurrent episodes of pleurisy 61) Which of the following is not associated with thyroid disease?
e) increased incidence of chronic respiratory disease in family a) dermatitis herpetiformis
members b) urticaria
c) porphyria cutanea tarda
51) Which of the following are not consistent with primary d) vitiligo
(spontaneous) bacterial peritonitis? e) alopecia areata
a) abdominal discomfort and fever
b) ascitic fluid neutrophil count of> 250x106 cells/L 62) All of the following are treatments for non-scarring alopecia except:
c) ascitic fluid WBC count of >500x106 cells /L a) spironolactone
d) multiple organisms on culture and sensitivity of ascitic fluid b) minoxidil
c) hair transplantation
52) Which finding is NOT frequently found in Chronic Myelogenous d) intralesional triamcinalone
Leukemia (CML)? e) finasteride
a) elevated WBCs
b) elevated vitamin B12 level 63) A 70 year-old woman presents with acute knee arthritis.
c) elevated LDH Radiographs show meniscal calcification (chondrocalcinosis).
d) translocation between chromosomes 9 and 14 Analysis of the synovial fluid reveals weakly positive
e) increased uric acid level birefringent rhomboid-shaped crystals. The crystals are
most likely:
53) Schistocytes on blood film examination are UNLIKELY to be seen a) monosodium urate
in which of the following: b) calcium hydroxyapatite
a) thrombotic thrombocytopenia purpura (TTP) c) cholesterol
b) thalassemia d) calcium pyrophosphate dihydrate
c) vasculitis e) dicalcium phosphate dihydrate (Brushite)
d) disseminated intravascular coagulation (DIC)
e) glomerulonephritis 64) Entamoeba histolytica is transmitted to humans by:
a) ingestion of infective eggs
54) All of the following can INHIBIT the absorption of ingested b) ingestion of cysts
non-heme iron EXCEPT: c) ingestion of animal tissue that contains the larva
a) alcohol d) penetration of the skin by infective larva
b) achlorhydria e) ingestion of adult form
c) phosphate (i.e. as found in milk)
d) phytates (i.e. as found in cereals) 65) Strongyloides spp. is transmitted to humans by:
e) antacids a) ingestion of infective eggs
b) ingestion of cysts
55) Which of the following is true of Myasthenia Gravis? c) ingestion of animal tissue that contains the larva
a) in patients older than 60, thymic hyperplasia is a common d) penetration of the skin by infective larva
etiology e) ingestion of adult form
b) often associated with thyroid disease
c) antibodies that are produced against acetylcholinesterase 66) The biosynthesis of fungal ergosterol is inhibited by:
d) associated with small cell lung carcinoma a) amphotericin B
b) griseofulvin
56) Which of the following is true with respect to proteinuria? c) flucytosine
a) all proteinuria is secondary to glomerular disease d) nystatin
> 2 g/24 h = nephrotic syndrome e) ketoconazole
b) is always abnormal and indicative of serious renal disease
c) it may be normal for an individual to have <150 mg per day of 67) A 55 year-old woman with asthma is on systemic steroids for one
proteinuria year. She develops a recent right-sided pleural effusion. She feels
d) if a patient has 1.5 g of protein in 24 h they must have unwell and tires easily. Aspiration reveals a turbid fluid, a high
tubular-interstitial disease. lymphocyte count, high LDH, low glucose, and a pH of 7.4. The
most compatible diagnosis is:
57) In which of the following conditions would one not expect a a) pulmonary embolism
Trans-Tubular Potassium Gradient greater than 4? b) empyema
a) primary hypoaldosteronism c) tuberculosis
b) acute vomiting d) subphrenic abscess
c) renin-secreting tumour e) pancreatitis
d) unilateral renal artery stenosis
e) Gordon syndrome
58) The most significant cause of morbidity in the elderly:
a) arthritis
b) dementia
c) heart disease
d) stroke
e) hearing impairment
10 Sample Questions Medicine Toronto Notes 2005

68) Diagnosis of acute symptomatic pulmonary embolism can be 74) Nitroglycerin administered sublingually may contribute to the
excluded when which of the following is normal? relief of myocardial ischemic pain by each of the following
a) chest x-ray mechanisms EXCEPT:
b) ventilation-perfusion lung scan a) coronary vasodilation
c) bilateral leg venograms b) decreased venous pooling resulting in increased cardiac
d) PaO2 and A-a O2 gradient preload
e) CT scan of the pulmonary arteries c) reduced systemic vascular resistance
d) reduced ventricular volume
69) In acute pyelonephritis, which of the following is most commonly
associated with bacteremic spread from a distant focus? 75) At what CD4 count are HIV patients at increased risk of
a) Escherichia coli developing PCP?
b) Proteus sp. a) CD4 count >500x106
c) Staphylococcus aureus b) CD4 count 200-499x106
d) Serratia sp. c) CD4 count <200x106
e) Enterococcus sp. d) CD4 count < 500x106
e) none of the above
70) A patient presents with a decreased level of consciousness and
visual difficulties. Bloodwork reveals an anion gap of 22 and an 76) An 80 year-old female with a 25% reduction in her GFR requires
osmolar gap of 24. Which of the following is most likely which of the following adjustments for a drug that is
responsible? predominantly cleared by the kidneys?
a) ethanol 1. 25% decrease in individual doses
b) salicylates 2. 25% increase in dosing intervals
c) renal tubular acidosis type I 3. increase overall fluid intake
d) methanol 4. decrease length of therapy by 25%
e) diabetic ketoacidosis a) all of the above
b) 1 and 3
71) In the treatment of Type I Diabetes, which of the following is true? c) 2 and 4
a) sulfonylureas are useful as an adjunctive therapy to insulin d) 2,3, and 4
b) most patients are adequately controlled with one type of e) 1 and 2
insulin (non-mixed) only
c) once diagnosed with Type I, patients must immediately be 77) Consequences of immobility include which ONE of the following:
assessed for retinopathy a) diarrhea
d) during periods of illness or infection, patients may require b) maintenance of muscle mass
additional insulin c) urinary retention
e) the most common initial presentation is visual disturbance d) pneumonia
e) hastened wound healing
72) Which of the following is associated with thyroid disease?
a) neurofibromatosis 78) Which of the following is FALSE in terms of falls in the elderly?
b) vitiligo a) they are the most common cause of mortality due to injury
c) erythema nodosum b) environment plays a significant role
d) pemphigus vulgaris c) fractures most commonly involve the humerus
e) icthyosis vulgaris d) age-related sensory changes make the elderly more
susceptible
73) Each of the following is a correct statement about COPD EXCEPT: e) fear of falling contributes to self-protection immobility
a) the type of emphysema associated with smoking is usually
centriacinar
b) clubbing is not a clinical feature
c) long-term oral steroids should be avoided
d) smoking cessation does not lead to improvement of
pulmonary function
e) the aim of supplemental O2 therapy is to provide relief of
shortness of breath
Toronto Notes 2005 Obstetrics and Gynecology Sample Questions 11

Obstetrics and Gynecology


1) A 26 year-old primigravida presents at 40 weeks in active labour 8) A one year-old female presents with vaginal bleeding. Vaginal
with contractions every 2 minutes. She is diagnosed as having a inspection reveals the presence of a multicystic grape-like lesion.
transverse lie with the back up. Which of the following would be The most likely diagnosis is?
the most appropriate next step? a) sexual abuse
a) start isoxuprine (Vasodilan) b) DES syndrome
b) perform an external version c) sarcoma botyroides
c) prepare for an immediate C-section d) clear cell adenocarcinoma
d) rupture membranes and then perform an internal version e) exposure to exogenous estrogen
e) none of the above
9) A 24 year-old G1P0 is seen in the Emergency Department. Her LMP
2) A 25 year-old primigravida at 34 weeks gestation is thought to be was 8 weeks ago. She is experiencing lower abdominal cramping
small for dates by her physician and is sent for a sonographic and heavy vaginal bleeding with clots. Examination reveals a soft
evaluation. The ultrasound shows the biparietal diameter to be abdomen with mild lower abdominal tenderness. On pelvic exam,
appropriate for 34 weeks gestation. The abdominal circumference the vagina is filled with blood and clots. The cervical os is opened
is appropriate for 30 weeks gestation. The head:abdominal and tissue is protruding. The uterus is enlarged to a 6 week size.
circumference ratio is < 1. The estimated fetal weight is <10th Which of the following is the most likely diagnosis?
percentile for 34 weeks gestation. The amniotic fluid is decreased. a) ectopic pregnancy
Which of the following is the most likely diagnosis? b) threatened abortion
a) symmetrical IUGR c) degenerating fibroid
b) asymmetrical IUGR d) placenta previa
c) congenital anomaly e) incomplete abortion
d) congenital infection
e) unknown gestational age 10) A 24 year-old female G1P0 is admitted to hospital at 34 weeks
gestation. The patient describes passing a large volume of clear fluid
3) A 32 year-old G5P4 presents with an 8 week history of amenorrhea per vagina for 36 hours. Examination reveals the following: temp
andsuggestive symptoms of pregnancy. Physical examination 38.5C, no uterine contractions, vague lower abdominal tenderness,
reveals an irregular, enlarged uterus of 16 weeks size. Ultrasound fetal heart rate 185 bpm. Lab investigations reveal: Hb 120, WBC
confirms thepresence of an 8 week viable pregnancy and a 19.0, + nitrazine test of vaginal fluid, + ferning of vaginal fluid.
multiple fibroid uterus. The correct management for this patient is: Which of the following is the most likely diagnosis?
a) termination of the pregnancy with elective myomectomy a) premature rupture of membranes
2 months later b) premature rupture of membranes and chorioamnionitis
b) termination of the pregnancy with concomitant myomectomy c) premature labour
c) prudent observation with elective C-section at term d) acute pyelonephritis
d) prudent observation anticipating probable vaginal delivery e) acute vaginitis of pregnancy
e) myomectomy and follow pregnancy in usual way
11) A 24 year-old female presents with abdominal pain. ß-HCG is
4) An infant is born. At one minute the heart rate is 120 per minute, negative. Pelvic ultrasound shows a 5 cm right ovarian cyst. You
respiratory effort is a good strong cry, muscle tone is active, reflex would:
irritability is absent, colour is pink with blue extremities. What is a) perform immediate laporotomy
the one minute Apgar score? b) perform immediate laproscopy
a) 5 c) aspirate the cyst under ultrasonographic guidance
b) 6 d) order a CBC and a CA125
c) 7 e) expectant management with repeat ultrasound in 8 weeks
d) 8
e) 9 12) The pathology report following a therapeutic abortion shows
Arias-Stella phenomenon (decidualization). The most appropriate
5) Which of the following is the easiest and most reliable way of next step in the management of this patient would be?
detecting a retained succenturiate placental lobe? a) repeat a pregnancy test
a) palpation of the uterus b) laporoscopy
b) inspection of the maternal side of the placenta c) reassure the patient
c) visualization of the cervix on speculum exam d) prescribe oral contraception
d) inspection of the fetal side of the placenta e) repeat the D&C
e) pelvic ultrasound
13) In a patient whose blood type is O-Rh negative and whose husband
6) Choose the correct statement regarding operative obstetrics: has O-Rh positive blood, Rh immune globulin (Rhogam) should be
a) a first degree tear involves skin and vaginal mucosa as well as given after all the following EXCEPT:
underlying fascia and muscle a) artificial rupture of membranes
b) delivery aided by vacuum extraction is typically of shorter b) full term delivery of a baby with O-Rh positive blood type
duration than if forceps are used c) amniocentesis
c) one absolute prerequisite for the use of forceps is that the d) spontaneous abortion
baby’s presenting part must be below the ischial spines e) therapeutic abortion
d) a third degree tear extends through the anal sphincter
e) none of the above 14) Risk factors for shoulder dystocia include all the following EXCEPT:
a) maternal obesity
7) A 36 year-old female presents with a mucopurulent vaginal b) macrosomia
discharge. Gram stain of a cervical swab shows gram negative c) maternal diabetes
diploccocci. Which of the following is correct? d) prolonged second stage of labour
a) if untreated this condition will likely resolve spontaneously e) outlet forceps delivery
b) the most likely diagnosis is Strep group B
c) appropriate treatment is Metronidazole vaginal cream
d) appropriate treatment is a single I.M. dose of 250 mg
of Ceftriaxone
e) Public Health will most likely need to be notified
12 Sample Questions Obstetrics and Gynecology Toronto Notes 2005

15) Fetal exposure to lithium in the first trimester has been strongly 25) A 55 year-old woman comes to the office to discuss menopause.
associated with: You tell her all of the following are a part of this stage EXCEPT:
a) alopecia a) hot flushes
b) cleft lip and palate b) vaginal dryness
c) congenital heart defects c) mood swings
e) limb defects d) insomnia
d) mental retardation e) decreased libido
16) The commonest type of bleeding encountered with uterine 26) All of the following features have been described in women with
leiomyomata is: androgen insensitivity syndrome EXCEPT:
a) post-coital spotting a) mammary aplasia
b) mid-cycle bleeding b) female phenotype
c) hypermenorrhea c) absence of wolffian duct structures
d) oligomenorrhea d) short vagina
e) post-coital staining e) absence of mullerian duct structures
17) Which of the following statements about estrogen therapy in 27) The commonest indication for amniocentesis is:
postmenopausal women is correct? a) chromosome anomaly of the mother
a) it is a major risk factor for breast cancer b) teenage pregnancy
b) it decreases hepatic triglyceride production c) advanced maternal age
c) it produces hypertension d) family history of Down syndrome
d) it protects against vertebral compression fractures e) family history of cystic fibrosis
e) it directly stimulates the proliferation of vascular endothelium
28) On colposcopy, the transformation zone:
18) Which event is matched with the correct timing? a) contains columnar epithelium
a) maternal serum screening test: 20-22 weeks b) is the area between original squamous epithelium and columnar
b) post-partum visit: 6 weeks epithelium
c) chorionic villus sampling: 4-6 weeks c) contains metaplastic epithelium
d) oral glucose challenge test: 14-16 weeks d) contains atypical blood vessels
e) Rhogam to Rh negative women: 12-14 weeks
29) Complications of cesarean section include each of the
19) Routine blood work during the first antenatal visit include all of following EXCEPT:
the following EXCEPT: a) hemorrhage
a) CBC b) infection
b) HIV c) laceration of the fetus
c) rubella titre d) subsequent rupture of the uterine scar
d) blood group and type e) inversion of the uterus
e) Rh antibodies
30) Regarding laparoscopy:
20) All of the following are cardiovascular adaptations to pregnancy a) it is contraindicated in patients who are menstruating.
EXCEPT: b) patients with intestinal obstruction are still able to undergo
a) cardiac output increases 33-45% laparoscopy
b) stroke volume increases 10-30% c) the most common indication for therapeutic laparoscopy is tubal
c) heart rate decreases 12-18 beats per minute sterilization
d) systolic blood pressure decreases 4-5 mmHg d) all of the above are incorrect
e) diastolic blood pressure decreases 8-15 mmHg
31) Causes of secondary amenorrhea include:
21) A couple are unable to conceive after 1 year. Of the following a) Turner syndrome
statistics regarding infertility, which are correct: b) anorexia
a) 10-15% of couples are infertile c) androgen insensitivity syndrome
b) only 40% of couples achieve pregnancy within 6 months of d) gonadal dysgenesis
trying e) imperforate hymen
c) only 20% achieve pregnancy within 1 year of trying
d) male factors are responsible for over half of infertility causes 32) Risk factors for cervical carcinoma includes all of the
e) infertility is labelled after failure to conceive within 2 years of following EXCEPT:
trying a) human papilloma virus type 16 and 18
b) early age at first intercourse
22) Vaginal discharge which is fishy in odor and associated with c) smoking
>20% clue cells on microscopy will not be associated with which of d) nulliparity
the following: e) multiple sexual partners
a) be due to gardnerella vaginalis overgrowth
b) can be treated with metronidazole or clindamycin 33) A 35 year-old G0P0 has severe pain during menses which radiates
c) with the addition of KOH may see hyphae or spores into the anal region. On bimanual exam, she has a small, retroverted
d) must be treated in all pregnant women, including uterus and tender nodules can be felt on palpation of the uterosacral
asymptomatic ligaments. Which of the following conditions does she most likely
e) is rarely associated with inflamed or itchy vulva have:
a) chronic PID
23) In the treatment of an ectopic pregnancy with methotrexate all of b) adenomyosis
the following are true EXCEPT: c) fibroids
a) the pregnancy must not have ruptured d) endometriosis
b) it must be less than 5cm in size e) uterine carcinoma
c) patient should be clinically stable
d) compliance and f/u are ensured 34) Which of the following statements is correct?
e) no fetal heart activity has been noted a) cervical carcinoma is the most common gynecological
malignancy in Canada
24) Which of the following is associated with breech presentations? b) lichen sclerosis is treated with estrogen cream
a) fetal heart sounds heard best in the upper abdomen c) colposcopy is required after a finding of mild cervical
b) brachial plexus injury dysplasia on a routine Pap smear
c) prolapse of the umbilical cord d) follicular cysts typically regress with the following cycle
d) increased fetal mortality e) leiomyomata are malignant tumours
e) all of the above
Toronto Notes 2005 Obstetrics and Gynecology Sample Questions 13

35) If a mother nurses her child, the time of reappearance of her 43) The appropriate next step in a woman found to have 1 abnormal
menses is uncertain but usually occurs after her delivery by: Pap smear showing mild dysplasia (CIN1) is:
a) 1-2 months a) repeat in 1 year's time
b) 2-3 months b) repeat in 4-6 months time with colposcopy if abnormal
c) 3-4 months c) send patient directly for colposcopy and LEEP excision
d) 4-5 months d) send for colposcopy after 1 abnormal Pap
e) > 5 months e) repeat Pap immediately and treat for HPV infection
36) Which of the following statements regarding malignant cervical 44) The commonest cause of disseminated intravascular
lesions is true? coagulation during pregnancy is:
a) 95% are squamous cell carcinoma a) amniotic fluid embolism
b) CA-125 levels to monitor treatment effectiveness are indicated b) abruptio placentae
c) the majority of lesions arise outside the transformation zone of c) placenta previa
the cervix d) missed abortion
d) they are not associated with HPV infection e) chorioamnionitis
e) treatment for stage 4 disease radical hysterectomy with
chemotherapy 45) Contraindications to vaginal delivery include each of the
following EXCEPT:
37) In endometriosis, the most common location for disease is: a) previous classical cesarean section
a) broad ligament b) squamous cell carcinoma in situ of the cervix
b) cul-de-sac c) total placenta previa
c) ovaries d) transverse lie with ruptured membranes
d) appendix e) primigravida breech with cephalo-pelvic disproportion
e) uterosacral ligament
46) In a differential diagnosis of third trimester bleed, which of the
38) All of the following increase the risk of developing ovarian cancer following should not be included:
EXCEPT: a) placenta previa
a) family history b) vasa previa
b) BCP c) abruptio placentae
c) nulliparity d) threatened abortion
d) late menopause e) marginal sinus hemorrhage
e) caucasian
47) A 41 year-old woman presents with obesity, hirsutism and
39) In the first 100 days of pregnancy, human chorionic gonadotropin oligomenorrhea, which of the following tests will give the least
titre doubles: useful information?
a) every day a) FSH
b) every week b) LH
c) every 2 days c) testosterone
d) every month d) TSH
e) every 2 weeks e) urine free cortisol
40) A young female enters your office wanting to start oral 48) With respect to androgen insensitivity syndrome, which of the
contraceptive pills. You remind yourself of the reasons she may following is true?
not start the pill which are: a) genotype is XY
a) impaired liver function b) breast development and uterus are absent
b) undiagnosed abnormal uterine bleeding c) serum testosterone is below normal male range
c) congenital hyperlipidemia d) secondary sex characteristics are male
d) past history thrombophlebitis e) the condition is autosomal recessive
e) all of the above.
49) The new selective estrogen receptor modulators have all of the
41) Which of the following statements regarding endometriosis is following benefits EXCEPT:
incorrect? a) no agonistic effect on breasts
a) theories proposed to explain the histogenesis of endometriosis b) provides estrogenic benefit on bone
include retrograde menstruation and coelemic metaplasia c) mimics estrogenic effect on cardiovascular status
b) CA-125 is often useful in making the diagnosis of d) no agonistic effect on uterine tissue
endometriosis e) provides relief of hot flushes associated with menopause
c) symptoms of endometriosis may include urinary frequency
and diarrhea 50) Regarding birth control, which of the following is false?
d) the differential diagnosis of endometriomas include a) it is contraindicated in women over age 35 who are smokers
hemorrhagic b) its efficacy may be decreased when taking antibiotics
corpus luteum cysts as well as neoplasms c) reduces the risk of ovarian carcinoma
e) the risk of endometriosis is several fold greater if there is a d) is associated with only a 10% failure rate
first degree relative with this condition e) can be associated with breast changes and weight gain.
42) A 55 year-old female presents with a "period-like" vaginal
bleeding for 5 days that stopped spontaneously. Your approach
to the problem would be to:
a) reassure the patient and manage expectantly
b) administer provera 10 mg od for 15 days
c) administer cyclic estrogen plus progesterone
d) obtain endometrial tissue
e) perform a pap smear plus endocervical curretage
14 Sample Questions Pediatrics Toronto Notes 2005

Pediatrics
1) A 6 month-old infant presents in the winter with fever, cough, 7) Which of the following investigations is most helpful in the
wheezing, tachypnea and decreased appetite. A chest assessment of a child presenting with an acute asthma attack?
radiograph shows hyperaeration and streaky perihilar infiltrates a) white cell count and differential
bilaterally. You diagnose bronchopneumonia. Which organism b) arterial blood gases
would most likely be causing this child's infection? c) chest x-ray
a) Chlamydia pneumoniae d) pulmonary function tests
b) Mycoplasma pneumoniae e) O2 sat. monitoring
c) Streptococcus pneumoniae
d) Haemophilus influenzae 8) A 6 year-old boy is brought to emergency with a 5 day history of
e) Respiratory syncytial virus fever, cough, and poor appetite, but no vomiting. On exam, he
appears unwell, is febrile, and has crusty nasal discharge and a
The following case pertains to questions 2 and 3: wet-sounding cough. Chest x-ray reveals a pulmonary infiltrate in
A 12 month-old girl is brought to the emergency department for the right middle lobe. He is admitted to hospital for pneumonia.
the second time in 2 days for vomiting and passage of 8 to 10 What is the best management for this child?
watery stools per day. a) observe for 24 hours with IV fluids only
b) observe for 24 hours with IV fluids and acetaminophen
2) Of the following, which provides the best estimate of the c) give oral amoxicillin and acetaminophen
patient’s volume deficit: d) give IV ampicillin and oral acetaminophen
a) weight change since the beginning of the illness e) give oral erythromycin and acetaminophen
b) hydration of mucous membranes, skin turgor, and
level of consciousness 9) Side effects of methylphenidate (Ritalin) therapy for hyperactivity
c) pulse, blood pressure, and peripheral capillary filling time include each of the following EXCEPT:
d) serum electrolytes a) anorexia
e) serum urea nitrogen and creatinine levels b) insomnia
c) tachycardia
3) The patient has lost 0.6 kg. She is moderately lethargic and has d) dependence (addiction)
dry mucous membranes and reduced skin turgor. Blood pressure e) growth suppression
is 80/40 mm Hg, and pulse is 120 per minute; capillary refill is
reasonably brisk. Lab studies reveal: sodium 131, potassium 4.8, 10) In comparing breast milk and formula, which of the following
chloride 101, bicarbonate 16 mEq/L, urea nitrogen 24 mg/dL and statements is NOT correct:
creatinine 0.6 mg/dL. The best strategy for managing this child is a) breast milk has a higher percentage of protein
to: b) breast milk has whey:casein ratio of 60:40
a) hospitalize for administration of IV fluid therapy c) breast milk contains leukocytes, complement and lysozymes
b) administer an oral rehydrating solution while the d) breast milk has a lower concentration of iron than iron
child is under medical supervision for 4-6 hours fortified formulae
c) instruct the parents about oral rehydration at home e) breast milk has an optimal calcium:phosphorus ratio of 2:1
d) hospitalize after giving 20 mL/kg of 0.9% saline IV
e) instruct parents on use of soy formula 11) Most umbilical hernias in children:
a) need strapping
4) Which of the following statements about Attention Deficit and b) resolve spontaneously
Hyperactivity Disorders (ADHD) is INCORRECT? c) require elective surgery
a) ADHD is more commonly seen in boys than girls d) require immediate surgery
b) deafness and seizure disorders must be excluded before e) are associated with a higher incidence of inguinal hernias
the diagnosis of ADHD is made
c) three characteristic features of ADHD are inattention, 12) A newborn male spits up his first feeding and develops bilious
hyperactivity, and mental retardation emesis with subsequent feedings. On physical exam he appears
d) ritalin is effective in the treatment of about 70% of cases of ill, has a scaphoid abdomen and absent bowel sounds.
ADHD Abdominal x-ray shows air in the proximal small bowel, but a
e) side effects of Ritalin include poor growth, insomnia, and tics paucity of air in the distal digestive tract. The most likely cause
for this infant's vomiting and clinical finding is:
5) A 6 year-old child is brought into the Emergency with his father a) antral web
because of a fractured arm. Although his father states that the b) choldochal cyst
child fell off his bike, the fracture does not seem consistent with c) Hirschsprung disease
this story. On inspection, you discover that the child seems to d) tracheoesophageal fistula
have a number of old bruises on his back and on his legs, some e) volvulus
of which seem reminiscent of finger markings. Which of the
following steps is not appropriate in this circumstance? 13) Breastfeeding is contraindicated if the mother has:
a) let the child go home with his father since you cannot really a) acute EBV infection
prove anything b) acute HepA infection
b) document the injuries that you saw with possible c) chronic HepB infection
photographs d) asymptomatic HIV infection
c) call the CAS and inform them e) none of the above
d) try to interview the child alone
e) do additional investigations (e.g. skeletal survey) 14) A 10 month-old child develops low-grade fever and sunburn-like
erythema over much of the body, but most prominently
6) A 14 year-old girl has refused to go to school 3 times in the past in the intertriginous areas. Within 36 hours, sheet-like
4 months. She says, "My tummy hurts," but she cannot point to desquamation is noted in the flexures and around the mouth.
where it bothers her. Her appetite is good, her bowel movements The mucous membranes are spared. Which of the following is
are normal, and she is sleeping well at night. She is a the most likely diagnosis:
healthy-looking girl with no abnormal findings on physical exam. a) Kawasaki disease
What is the most likely diagnosis? b) staphylococcal scalded skin syndrome
a) a benign abdominal mass compressing her duodenum c) Stevens-Johnson syndrome
b) diaphragmatic hernia d) toxic epidermal necrolysis
c) somatoform disorder e) toxic shock syndrome
d) gastroesophageal reflux disease
e) early peptic ulcer disease
Toronto Notes 2005 Pediatrics Sample Questions 15

15) By one year of age, a child is able to have all of the following 23) When prophylactic antibiotic therapy is used for tooth
foods except: extraction in a patient with a ventricular septal defect:
a) breast milk a) therapy is started 24 hours prior to the procedure
b) peeled fruits b) a throat swab should be taken prior to the procedure
c) cheese c) a second generation cephalosporin (cefuroxime) is the
d) peanuts therapy of choice
e) eggs d) therapy is given for one week following the extraction
e) erythromycin is the drug of choice for those allergic to
16) Central cyanosis in the newborn infant is most often caused by: penicillin
a) congenital heart disease
b) lung disease 24) Which of the following statements about stuttering in a
c) central nervous system disease 4 year-old is incorrect:
d) methemoglobinemia a) it is characterized by intermittent difficulty in producing a
e) hypoglycemia smooth flow of speech
b) it is more than 3 times more frequent in girls than in boys
17) In a premature infant who is suspected of having necrotizing c) it is exacerbated by anxiety
enterocolitis (NEC), each of the following is correct EXCEPT: d) more than 30% of children who stutter recover
a) Septicemia is associated with an increased risk of NEC spontaneously
b) NEC is thought to be caused by systemic hypertension e) none of the above
c) the finding of air in the portal vein indicates severe illness
d) respiratory distress increases the risk of NEC 25) A woman who is positive for hepatitis B surface antigen
e) apgar scores inversely correlate with the risk of NEC (HBsAg), but negative for hepatitis B antigen (HBeAg),
delivers at term. What would be the best management for
18) Which of the following is the most significant risk factor for the this woman's infant?
development of childhood asthma: a) administer gamma globulin intramuscularly immediately
a) family social background and at 1 month of age
b) parental asthma b) administer hepatitis B (HB) vaccine immediately and at 1
c) stress in the family month and 6 months of age
d) parental smoking c) administer hepatitis B immune globulin (HBIG) if cord
e) presence of pets in the house blood is positive for HbsAg
d) administer HBIG and HB vaccine immediately, and HB
19) Which of the following statements about sickle cell disease is true? vaccine again at 1 month and 6 months of age
a) all patients with sickle cell disease have a homozygous HbSS e) advise mother that breastfeeding is contraindicated
genotype
b) sickle cell disease causes a severe chronic anemia that is not r 26) Regarding sexual abuse of children, each of the following
outinely transfusion dependent statements is true EXCEPT:
c) patients have increased susceptibility to infection by a) no genital injury is found in the majority of patients
nonencapsulated organisms b) father-daughter incest is more common than brother-sister
d) patients usually present with sickle cell crises within one month c) most assailants are unknown to the victim
of age d) laboratory findings usually show no presence of sperm
e) splenic dysfunction usually does not occur until the child enters e) half of the abused children come from single parent
his/her teens families
20) Which of the following is false about physiological jaundice? 27) The viral infection most likely to cause CNS involvement and
a) it rarely presents before the age of 24 hours focal neurological findings is :
b) it may cause kernicterus a) coxsackievirus
c) in premature infants it may persist for 3-4 weeks b) herpes simplex
d) it is mainly due to temporarily impaired hepatic clearance of c) enterovirus
bilirubin d) rabies
e) it does not always require treatment with phototherapy e) rhinovirus
21) Which of the following statements is true? 28) Which of the following predisposes infants to chronic otitis media?
a) cancer is the second most common cause of death in children a) bottle-feeding in upright position
b) Hodgkin's lymphoma is the most common childhood cancer b) abnormal tympanic membrane formation
c) hyperploidy in leukemic cells is a poor prognostic indicator c) environmental factors such as daycare and passive smoking
d) Wilm's tumor is rarely associated with other congenital d) allergies
abnormalities e) none of the above
e) neuroblastomas usually occur in late adolescence
29) A 13 month-old infant boy has chronic diarrhea, poor appetite,
22) A 5 year-old girl with hypogammaglobulinemia and absent irritability and growth failure. He had been well previously,
immunoglobulin A (IgA) receives infusions of immune developing normally until diarrhea began at 3 months of age.
globulin monthly. Shortly after her most recent infusion Findings include weight loss less than 5th percentile and length
began, she developed hypotension, wheezing, and several at 25th percentile, cachectic appearance, wasted extremities and
urticarial lesions. Which of the following best explains her protuberant abdomen. His labs are: albumin 23, protein 40.
reaction? Stool positive for reducing sugars and negative for enteric
a) anaphylactic reaction due to IgE anti-IgA antibodies pathogens and ova and parasites. These findings are most
b) Gram-negative sepsis due to contaminated immune globulin consistent with:
c) idiosyncratic reaction due to rapid infusion of immune a) celiac disease
globulin b) cow milk allergy
d) serum sickness reaction from foreign serum in the immune c) Crohn disease
globulin d) Cystic Fibrosis
e) transfusion reaction due to ABO incompatibility e) none of the above
16 Sample Questions Pediatrics Toronto Notes 2005

30) All infants less than 3 months of age who have fever and no 38) An 8 year-old boy has had paroxysmal abdominal pain since his
localizing signs should have all of the following evaluations parents separated 6 months ago. Which of the following
EXCEPT: symptoms would support and organic basis for his disease?
a) blood cultures a) headaches accompanies the pain
b) stool cultures b) pain located in the periumbilical region
c) chest x-ray c) pain awakens child at night
d) CBC (with differential) d) symptoms last < 1hr
e) urine culture e) none of the above
31) All the following statements regarding transient tachypnea of the 39) The most common cause of chronic diarrhea in a
newborn (TTN) is true EXCEPT: 6 month - 36 month old child is:
a) infants born by C-section are at increased risk for developing a) toddler’s diarrhea
TTN b) disaccharidase deficiency
b) residual pulmonary function disability is common among c) enteric infection
infants who have TTN d) malabsorption
c) the incidence of TTN is higher than Respiratory Distress e) protein intolerance
Syndrome (RDS) among term infants
d) TTN shows marked improvement with 12-24 hrs 40) Which of the following is NOT required for a diagnosis of
e) chest x-ray findings consisntent with TTN include increased juvenile rheumatoid arthritis?
pulmonary vasculature and fluid in the fissures a) arthritis in at least one joint
b) arthritis lasting for at least 6 weeks
32) Common signs of heart failure in children include all of the c) positive rheumatoid factor
following EXCEPT: d) onset before the age of 16
a) decreased exercise tolerance e) other causes of arthritis excluded
b) fatigue
c) failure to thrive 41) A child presents with bilateral shin pain. Which of the following
d) frequent URTIs suggests that this is NOT growing pains?
e) peripheral edema a) pain is poorly localized
b) pain awakens the child at night
33) Antibiotic prophylaxis against infective endocarditis is required for c) no fever or rash
all of the following EXCEPT: d) pain abates with reassurance and massage
a) rheumatic valve lesions e) child may limp in the morning from stiffness
b) prosthetic heart valves
c) isolated secundum ASD 42) Which of the following statements about precocious puberty is
d) pacemaker leads true?
e) PDA a) the cause is never found in most cases
b) the children are always shorter than average
34) Causes of microcytic anemia include all of the following EXCEPT: c) the incidence is the same for males and females
a) excessive cow's milk intake d) ketoconazole is the treatment of choice for females
b) iron deficiency e) all girls with breast development before age 10 require
c) folic acid deficiency karyotyping for Turner syndrome
d) thalassemias
e) lead poisoning 43) All of the following groups are contraindicated to receive the oral
polio vaccine EXCEPT:
35) The latest time after the onset of GABHS pharyngitis that initiation a) pregnant women
of penicillin therapy can be expected to prevent acute rheumatic b) asymptomatic HIV-positive children
fever is: c) household contacts of immunodeficient children
a) 24 hrs d) unimmunized adults at future risk of being exposed to the
b) 48 hrs polio virus
c) 96 hrs e) children with agammaglobulinemia
d) 9 days
e) 2 weeks 44) Which of the following steps is NOT indicated in the
management of croup?
36) Fetal tricuspid valve abnormalities are associated with a) keeping the child calm
maternal use of which of the following: b) hydration
a) lithium c) antipyretics
b) warfarin d) antibiotics
c) chloramphenicol e) humidified oxygen
d) estrogen
e) heroin 45) Which of the following pairs shows the CORRECT stage of
normal development?
37) Which of the following is most likely to provide the basis for a) 6 months - pulls to stand
making the correct diagnosis in an infant or child who has failure to b) 12 months - pincer grasp
thrive (FTT) ? c) 18 months - handedness
a) blood chemistries d) 2 years - draws a cross
b) cultures e) 3 years - tells a story
c) history and physical
d) radiograph studies 46) Which of the following is NOT characteristic of a functional
e) none of the above murmur?
a) pansystolic murmur
b) murmur varies with position
c) variably split S2
d) murmur becomes louder with fever
e) no extra clicks
Toronto Notes 2005 Pediatrics Sample Questions 17

47) Which of the following is the most common congenital heart 55) A 7 day-old breastfed infant born at term has had decreased
lesion in children? appetite, irritability, and vomiting for 24 hours. On physical
a) ASD examination, the infant appears listless. Respiratory rate is
b) VSD 40/min; heart rate, 160/min; and blood pressure, 68/38 mm Hg.
c) PDA he skin and sclerae are icteric but no other abnormalities are
d) Tetralogy of Fallot noted. Laboratory studies reveal: hemoglobin 120 g/L; total
e) coarctation of the aorta bilir bin, 270 umol/L; and direct bilirubin, 135 umol/L.
Urinalysis is negative for reducing substances. Which of the
48) Choose the INCORRECT statement about breastfeeding: following is the most likely diagnosis:
a) for healthy term babies, breastfeeding is recommended over a) bacterial sepsis
formula feeding b) blood group incompatibility
b) breastmilk contains more amino acids than cow’s milk c) breast milk jaundice
c) exclusively breastfed babies should be supplemented with d) hypothyroidism
iron after 6 months of age e) intrauterine infection
d) assessment of adequate intake can be done by counting
the number of wet diapers in a day 56) Each of the following statements about rotavirus gastroenteritis
e) an HIV-positive mother should not breastfeed in children is correct except:
a) it usually occurs in the winter months
49) All of the following are characteristics of a child with Pervasive b) it is self-limited
Developmental Disorder (PDD) EXCEPT: c) it is often associated with respiratory symptoms
a) abnormal speech pattern d) it is often accompanied by fever and vomiting
b) stereotypic behaviours, such as head-banging or hand-flapping e) it is often associated with grossly bloody diarrhea
c) abnormal cognitive function
d) consuming interest in one topic or activity 57) The most common cause of deterioration of diabetic control
e) tendency to reciprocate in peer interactions in an 8 year-old boy is:
a) change in diet
50) All of the following are appropriate in the initial assessment of a b) lack of compliance
child with failure to thrive EXCEPT: c) emotional stress
a) diet history d) acute infection
b) social history e) change in exercise level
c) measurement of height, weight and head circumference
d) growth hormone levels 58) A child born at 32-weeks gestation is brought to your office at
e) bone age x-ray 2 months of age for a routine check-up. The child has an
upper respiratory tract infection but is afebrile. What decision
51) Which of the following statements about a child with short stature is would you make regarding the child's first vaccination?
CORRECT? a) wait until the child is two months older to administer the
a) the bone age is delayed in a child with constitutional growth first vaccination
delay b) wait until the child is clinically well and then vaccinate
b) height crosses a major percentile line on the child’s growth chart c) administer the first vaccination
c) weight is more affected than height in a child with an endocrine d) consult a pediatrician
deficiency e) none of the above
d) karyotyping is part of the routine investigation of all children
with short stature 59) The most common cause of hemoptysis in a 6 year-old boy is:
e) growth hormone replacement is helpful in a child with familial a) cystic fibrosis
growth delay b) pneumonia
c) foreign body
52) An 8 year-old boy who has been continent of urine between the d) tracheobronchitis
ages of 4 and 5 years comes in because of recent onset of e) neoplasm
enuresis. Which of the following statements is false?
a) it may be precipitated by a family crisis 60) A 15 month-old child has spastic cerebral palsy. Which of the
b) it does not require a specific treatment following is most suggestive of perinatal asphyxia as the cause
c) it can lead to poor self esteem for this condition:
e) it responds best to individual psychotherapy a) evidence of periventricular calcificatons at birth
e) it can be secondary to diseases such as diabetes mellitus b) microcephaly at birth
c) recurrent seizures in the newborn period
53) Which of the following is NOT correct about the pertussis d) retinopathy of prematurity
vaccine: e) rigidity and spasticity
a) delaying the primary series until the age of 1 year will not
reduce the risk of a seizure 61) Which of the following findings is least consistent with a
b) the commonly quoted risk of 1:330,000 of brain damage has diagnosis of congenital deafness in a 16 month-old infant:
no basis in fact a) smiling responsively at 1 month
c) it can cause febrile seizures b) cooing at 2 months
d) it may cause infantile spasms or sudden infant death c) babbling at 3 to 6 months
syndrome d) imitating speech sounds at 9 months
e) the risk of other forms of neurologic illness eg. transverse e) no clearly enunciated words at 15 months
myelitis is extremely small
62) Which of the following genetic diseases is incorrectly
54) The most common cause of apnea in infants less than linked with its transmission pattern:
6 months is: a) Duchenne muscular dystrophy = sex-linked recessive
a) encephalitis b) pyloric stenosis = polygenetic inheritance
b) seizure disorder c) cerebral palsy = polygenetic inheritance
c) cardiac arrhythmia d) sickle cell disease = autosomal codominant
d) milk allergy e) color blindness = autosomal recessive
e) gastroesophageal reflux
18 Sample Questions Pediatrics Toronto Notes 2005

63) Each of the following is an indication for tonsillectomy except: 70) During the newborn period, physical characteristics suggestive
a) recurrent episodes of otitis media of prematurity include each of the following except:
b) peritonsillar abscess a) a prominent diastasis of the rectus muscles
c) carcinoma of the tonsil b) scant ear cartilage
d) hypertrophy with airway obstruction c) absent breast buds
e) recurrent (at least six) episodes of group A Strep d) thin skin
tonsillitis within one year e) hypotonic posture at rest
64) A healthy, full-term, breastfed infant is jaundiced at 24 hours of 71) A 5 year-old child is diagnosed as having juvenile rheumatoid
age. The indirect serum bilirubin level is 210 umol/L. The arthritis. Which of the following is the least likely side effect of
mother is blood type O, Rh positive. Which of the following is therapy with a non-steroidal anti-inflammatory drug:
the most appropriate step: a) abdominal pain
a) culture the blood b) lymphadenopathy
b) determine glucose-6-phosphate dehydrogenase activity c) tinnitus
c) interrupt breast feeding d) anemia
d) obtain a reticulocyte count e) rash
e) obtain blood type and Coombs test
72) Which of the following statements about tumours in childhood
65) A 6-week-old male infant develops recurrent episodes of is false:
vomiting with dehydration. Each of the following would be a) ALL is the most common malignancy
consistent with a diagnosis of congenital hypertrophic pyloric b) brain tumours are the most common solid tumours
stenosis except: c) malignancy is the third most common cause of death after
a) non-bile stained vomitus after feeding accidents and homicides
b) a metabolic acidosis and hyperkalemia d) neuroblastoma spontaneously regresses
c) a history of increasing constipation e) blacks and whites do not have equal incidence of tumours
d) good appetite
e) visible epigastric peristaltic waves 73) Which of the following is a characteristic chest x-ray
appearance of respiratory distress syndrome (RDS):
66) Each of the following statements regarding AIDS in infants and a) air bronchograms
children is correct except: b) fluid in fissure
a) the risk of nosocomial transmission is low c) patchy infiltrate
b) function of both T and B lymphocytes is impaired d) atelectasis
c) hepatosplenomegaly is a common associated finding e) enlarged heart
d) poor growth occurs frequently
e) chronic pneumonitis is a rare complication 74) An 8 year-old boy presents with sudden onset of pain in the
distal femur. His parents recall that he was hit with a hockey
67) A 14 year-old boy is concerned about his height. Physical stick during a street hockey game last week. The most likely
examination reveals Tanner stage II pubic hair and genital diagnosis is:
development. Height and weight are at the 10th percentile for a) osteomyelitis
age. Which of the following is the most accurate statement b) fracture of the distal femur
about his growth: c) juvenile rheumatoid arthritis
a) he is taller than the average 14 year-old girl d) septic arthritis
b) he will be taller than his peers by age 16 e) Blount’s disease
c) he has not reached his peak height velocity
d) he will stop growing by age 17 75) The most likely diagnosis in a 3 month-old with a left flank
e) his adult height will be at the 10th percentile mass is:
a) renal cell carcinoma
68) A 3 year-old girl has a urinary tract infection and receives b) Wilm’s tumor
appropriate antibiotic therapy. Following treatment, c) neuroblastoma
ultrasonography and voiding cystourethrography reveal normal d) renal stones
findings. Which of the following would be the best e) polycystic kidney disease
management of this patient's urinary tract problem?
a) administrate polyvalent vaccine against multiple enteric 76) Which of the following congenital anomalies usually resolves
organisms spontaneously:
b) begin daily prophylaxis now and treat each future episode a) club foot
of infection b) genu varus
c) obtain intravenous pyelography c) tibial torsion
d) treat each future episode of infection, but withhold d) flexible flat feet
prophylaxis e) genu valgus
e) none of the above
77) Which of the following is the most important factor in
69) A 10 year-old boy with cough, fever and night sweats has a determining whether a child will die of measles:
positive skin test for tuberculosis. A chest x-ray reveals minimal a) gender
hilar adenopathy and a small pulmonary infiltrate. The current b) age
recommended treatment is: c) nutritional status
a) isoniazid alone d) muscle mass
b) isoniazid plus rifampin and pyrazinamide e) fitness level
c) isoniazid plus rifampin and streptomycin
d) rifampin and corticosteroids
e) rifampin and ethambutol
Toronto Notes 2005 Pediatrics Sample Questions 19

78) At 36 hours of age a full-term infant has not yet passed 81) A 1 year-old child with a rapid pulse, slightly low blood
meconium. Physical examination reveals jaundice and pressure, cool extremities, oliguria and dry oral mucosa should
abdominal distention. Radiographs of the abdomen show be considered:
dilated loops of bowel and calcifications in the scrotum. A a) 5% dehydrated
meconium plug is passed after rectal examination. Which of b) 10% dehydrated
the following would be the most definitive diagnostic test to c) 15% dehydrated
order: d) 20% dehydrated
a) barium enema
b) sweat chloride test 82) All of the following are indications for tympanostomy tubes
c) ultrasound of the abdomen ± adenoidectomy in a child except:
d) urine calcium excretion a) persistent 3 month effusion
e) voiding cystourethrogram b) failure of prophylactic medications
c) associated hearing loss
79) Organic causes in the differential diagnosis of chronic d) recurrent acute otitis media
abdominal pain includes all of the following except:
a) UTI 83) A 2 year-old with fever is seen in hospital following a
b) lactose intolerance generalized seizure. He also has foul smelling, blood-tinged
c) chronic giardiasis diarrhea. The culture of the stool is most likely to grow:
d) chronic appendicitis a) Campylobacter species
e) inflammatory bowel disease b) E. coli
c) Entamoeba histolytica
80) Which of the following problems is most likely to occur in a d) Salmonella species
12 month-old infant who began taking whole cow milk at e) Shigella species
6 months of age:
a) increased frequency of otitis media
b) wheezing episodes
c) nasal congestion
d) decreased serum ferritin level
e) diaper dermatitis
20 Sample Questions Psychiatry Toronto Notes 2005

Psychiatry
1) Monoamine oxidase inhibitor drugs are used in the treatment of 9) A 32 year-old engineer has been uncharacteristically active for
depression because they increase synaptic levels of: several weeks. He spends most of his time at work and gets
a) gamma-aminobutyric acid (GABA) little sleep. He has told another engineer that he is involved “in
b) histamine a research project that will earn me the Nobel Prize”.
c) acetylcholine Expensive research equipment keeps arriving at his office. The
d) norepinephrine engineer is irritable, and it is hard to hold his attention. A
e) somatostatin classmate from graduate school recalls that the patient
behaved in a similar manner twice during stressful periods at
2) Neuropsychological effects of hallucinogens may include all of the school.
following EXCEPT: Longterm drug therapy for this patient would likely include:
a) miosis a) haloperidol
b) tremor b) valproic acid
c) hyper-reflexia c) clozapine
d) uncoordination d) ascorbic acid
e) blurred vision e) chlordiazepoxide
3) Cocaine withdrawal can include all of the following EXCEPT: 10) Elderly male depressives typically present with all of the
a) “Crash” sleep following EXCEPT:
b) anergia a) importuning
c) anhedonia b) anxiety
d) euphoria c) weight loss
e) continued craving d) little suicide risk
e) insomnia
4) Alcohol withdrawal includes all of the following EXCEPT:
a) autonomic hyperactivity 11) From among the drugs listed below, which would be the cause
b) tremor for most concern in an overdose:
c) starts within 2-4 hours after prolonged drinking a) paroxetine (SSRI)
d) nausea b) amitriptyline (tricyclic)
e) irritability c) diazepam (benzodiazepine)
d) chlorpromazine (phenothiazine)
5) Which would not be considered a risk factor for suicide in patients e) fluoxetine (SSRI)
presenting with suicidal ideation:
a) substance abuse 12) Which of the following statements about schizophrenia is false?
b) male gender a) male schizophrenics experience their first psychotic
c) lack of social supports episode at a younger age than women
d) unsuccessful attempt at suicide in the past b) male schizophrenics are more frequently hospitalized than
e) childless marriage female schizophrenics
c) compared to young female schizophrenics, young male
6) A 54 year-old man has become forgetful, preoccupied, schizophrenics are at increased risk of movement disorders
withdrawn, irritable and dishevelled. His physical examination secondary to neuroleptics
was normal. The patient had been with his company for d) in women, the symptoms of schizophrenia tend to worsen
twenty-two years and was considered an excellent employee. after menopause
Which of the following is the most likely diagnosis: e) all of the above statements are true
a) multi-infarct dementia
b) hypothyroidism 13) A 29 year-old school teacher who lives alone is brought to the
c) schizophrenia emergency room because she has become increasingly suspicious,
d) alcoholism hyperactive, and anorexic over the past two days. She believes
e) major depression that “people in the neighbourhood are out to get me”. She has
not slept in 2 nights. She reports seeing snakes crawling on the
7) Which of the following is correct about depression in wall. Based on this information, the most likely diagnosis of the
children: woman’s problem is:
a) family therapy should be avoided because it scapegoats a) anorexia nervosa
a child who is already vulnerable b) cocaine withdrawal
b) symptoms may manifest as antisocial behaviour c) paranoid personality
c) antidepressants generally are not effective in children d) psychostimulant abuse
d) the suicide rate in children aged 8-13 is higher than e) shared paranoid disorder
it is in older adolescents
e) depression in children has been shown to be a prodrome 14) Anti-alpha-1-adrenergic blockade causes:
to the later development of schizophrenia a) nausea
b) constipation
8) All of the following are classified as paraphilias EXCEPT: c) orthostatic hypotension
a) fetishism d) dry mouth
b) homosexuality e) drowsiness
c) exhibitionism
d) sexual sadism 15) The following are common side effects of SSRIs EXCEPT:
e) transvestism a) Headache
b) Sexual dysfunction
c) Vomiting
d) Anorexia
e) Orthostatic hypotension
Toronto Notes 2005 Psychiatry Sample Questions 21

16) Clozapine is the neuroleptic of choice for schizophrenia when: 23) Hypnosis has been used successfully in all of the following
a) the patient shows no evidence of tardive dyskinesia conditions EXCEPT:
b) the patient has not improved with adequate a) pain
dosages for 6 weeks of 3 other antipsychotics b) phobia
c) the patient is under fifteen years of age c) paranoia
d) the patient has a WBC count of >10 000 d) anxiety
e) the patient is very sensitive to anticholinergic side effects e) smoking
17) Frequent conditions appearing comorbidly with ADHD include: 24) Cognitive therapy helps to correct which of the following
1) oppositional defiant and conduct disorders cognitive distortions:
2) anxiety disorder a) depersonalization
3) learning disabilities b) psychotic thinking
4) tic disorders c) over-generalizations
5) language disorders d) selective inference
e) hallucinations
Which are correct:
a) only 1 is correct 25) The most frequently reported side effect of tricyclic
b) 1, 2, and 3 are correct antidepressants is:
c) 2 and 4 are correct a) peripheral neuropathy
d) all are true b) photosensitivity
c) agranulocytosis
18) A 35 year-old man presents to the emergency room with d) jaundice
suicidal ideation. He describes significant stress due to e) dry mouth
recent job loss and financial hardship. Further inquiry reveals
a history of repeated job loss, fraud charges, and frequent 26) A high risk of suicide is associated with which one of the
arm slashing to decrease stress. The man was not disruptive following factors:
as a child. The most likely diagnosis is: a) female aged less than 30 years
a) borderline personality disorder b) married male less than 30 years
b) antisocial personality disorder c) public setting
c) adjustment disorder d) secondary gain from attempt
d) dysthymic disorder e) single male aged more than 60 years
e) schizophrenia
27) A fixed unalterable belief that is false in its content and in its
19) Which of the following statements concerning anorexia social and cultural setting is called:
nervosa and bulemia nervosa is false: a) an illusion
a) patients with either of these eating disorders are b) a hallucination
preoccupied with weight, food, and body shape c) a delusion
b) bulemia nervosa is more prevalent than is anorexia nervosa d) agnosia
c) both of these eating disorders are more common in e) paranoia
females than in males
d) bulemia nervosa often presents earlier in adolescence than 28) Criteria for involuntary hospitalization of a suicidal patient
does anorexia nervosa may include:
e) bulemic symptoms may occur in both bulimia nervosa and a) acute risk of harm to themself
anorexia nervosa b) inability to care for self
c) acute risk of harm to others
20) A pattern of unstable but intense interpersonal relationships, d) (b) and (c)
impulsivity, inappropriately intense anger, identity disturbance, e) (a), (b), and (c)
affective instability, and problems with being alone suggest a
diagnosis of: 29) Each of the following statements about affective disorders is true
a) antisocial personality disorder EXCEPT:
b) narcissistic personality disorder a) patients with bipolar disorder show roughly the same
c) histrionic personality disorder frequency of positive family history as do patients with
d) schizoid personality disorder unipolar disorder
e) borderline personality disorder b) major depressive illness is more common in women than in
men
21) The criteria for diagnosis of a factitious disorder include: c) bipolar affective disorder is far less common than unipolar
a) intentional production or feigning of physical signs or affective disorder
symptoms d) there is a different response to lithium in unipolar and bipolar
b) absence of secondary gain disorder
c) possibility of economic gain e) imipramine is more likely to produce hypomania in bipolar
d) (a) and (b) patients than in unipolar patients
e) (a) and (c)
30) A major depressive disorder is diagnosed only when the affective
22) A patient with a fear of heights is brought to the top of a tall disturbance has existed for at least :
building and required to remain there as long as necessary a) two weeks
for the anxiety to dissipate. This is an example of: b) four weeks
a) graded exposure c) six weeks
b) participant modelling d) eight weeks
c) positive reinforcement e) twelve weeks
d) flooding
e) relationship therapy
22 Sample Questions Psychiatry Toronto Notes 2005

31) A 30 year-old man presents in emergency with right lower quadrant 40) Which of the following is best treated with high dose
abdominal pain. His wife reports that he had been drinking heavily benzodiazepines:
in response to marital problems and had never had such pain a) schizophrenia, catatonic type
before. Appendicitis was diagnosed and an appendectomy was b) major depression
successfully performed. Four days later the patient was anxious, c) generalized anxiety disorder
restless, unable to sleep and claimed his wife was a stranger trying d) delirium tremens
to harass him. The likeliest diagnosis is: e) psychogenic amnesia
a) paranoid reaction
b) delirium tremens 41) Personality types predisposed to depression include which
c) mania one of the following:
d) schizophreniform reaction a) dependent
e) post-operative delerium b) antisocial
c) schizoid
32) Which of the following symptoms of schizophrenia responds best to d) paranoid
anti-psychotic medication: e) schizotypal
a) anti-social behavior
b) flat affect 42) Which of the following neurological symptoms can be produced
c) paranoid delusions by antipsychotic drugs?
d) lack of motivation a) akathesia
e) all of the above b) shuffling gait
c) oculogyric crisis
33) A 53 year-old housewife presents with depression marked by early d) tremor at rest
morning wakening, diminished energy and poor concentration. She e) all of the above
is treated with supportive psychotherapy and celexa
20 mg. After four weeks she is no better. You would next: 43) Absolute contraindications to ECT include:
a) reassure her that antidepressants take 5-6 weeks to be a) pregnancy
effective b) recent myocardial infarction
b) begin intensive psychotherapy c) fractured pelvis
c) switch to a different SSRI d) brain tumor
d) increase the dose to 40 mg e) all of the above
e) (a) and (d)
44) In the elderly delirium may be produced by the use of:
34) Hypertensive encephalopathy is a serious complication of treatment a) neuroleptics
with: b) tricyclic antidepressants
a) phenothiazines c) antiparkinsonian agents
b) tricyclic antidepressants d) minor tranquilizers
c) lithium carbonate e) all of the above
d) MAOI antidepressants
e) benzodiazepines 45) Which of the following statements about simple phobia is correct?
a) it responds well to simple reassurance
35) Electroconvulsive therapy (ECT) is a treatment in b) it responds well to individual psychotherapy
which one of the following conditions? c) it responds well to benzodiazepines
a) obsessive compulsive disorder d) it responds well to relaxation and desensitization
b) paranoid schizophrenia e) all of the above
c) generalized anxiety disorder
d) acute mania 46) Toxicity due to lithium carbonate is associated with all
e) major depression with psychotic features of the following EXCEPT:
a) nausea
36) Tardive dyskinesia is: b) serum lithium level of 0.1 mEq/L
a) an acute extrapyramidal side effect of antipsychotic c) tremulousness
b) an acute anticholinergic side effect of tricyclic antidepressants d) convulsions
c) a type of Parkinson’s disease
d) a side effect of excessive ECT treatments 47) Which of the following investigations need not be performed
e) a long term complication of chronic antipsychotic before starting treatment with lithium carbonate?
administration a) serum creatinine
b) serum electrolytes
37) Which of the following symptoms is most commonly found in c) thyroid function studies
schizophrenia? d) serum bilirubin
a) depressed mood e) all of the above
b) flight of ideas
c) elevated mood 48) All of the following are common side effects of tricyclic
d) delusional fears antidepressants EXCEPT:
e) thought insertion a) dry mouth
b) constipation
38) Patients with conversion disorders will show each of the following c) Parkinsonian tremor
EXCEPT: d) tachycardia
a) “la belle indifference”
b) severe depression 49) Which of the following features are seen in dementia due to
c) loss of special sense function cerebral arteriosclerosis:
d) secondary gains a) “patchy” amnesia for remote events
e) paralysis of voluntary muscles b) more common in women than men
c) death occurs frequently due to cerebrovascular
39) The syndrome of delirium tremens is associated with each of the accidents
following EXCEPT: d) increased “senile plaque” count
a) rapidly fluctuating level of consciousness e) all of the above
b) dehydration
c) visual hallucinations
d) renal failure
e) polyneuropathy
Toronto Notes 2005 Psychiatry Sample Questions 23

50) The Theory of Behavioral Therapy employs which of the following 53) A young mother is very focused on the health of her
concepts: 16 month-old. She keeps her house immaculate for fear that
a) reaction formation dirt will harm her baby, she checks the lock on the door at
b) reinforcement least ten times before retiring to bed, and she has to get up
c) imprinting and check that her child is still breathing at least 3 times
d) autosuggestion every night. She knows that her fears are irrational but
e) sublimation persists with these behaviors. The most likely diagnosis is:
a) paranoid delusions not otherwise specified
51) Which of the following represents a contraindication for b) post-partum depression
psychoanalysis? c) obsessive compulsive disorder
a) the existence of transference feeling toward the analyst d) generalized anxiety disorder
b) the existence of countertransference feeling in the analyst e) paranoid personality disorder
c) the resistance to change by the patient
d) preoccupation with resolving an acute crisis situation 54) A thorough assessment for the presence/absence of alcohol
e) the existence of an underlying neurotic style of withdrawal should include questions about all of the following
behaviour EXCEPT:
a) nausea and vomiting
52) Which of the following is least helpful in treating depression? b) mood
a) phenelzine c) difficulty walking (ataxic gait)
b) imipramine d) visual disturbances
c) lithium carbonate e) tremulousness
d) lorazepam
e) all of the above 55) Which of the following will be least helpful with respect to
distinguishing delerium from dementia:
a) disorientation at night
b) duration of disorientation
c) fluctuating level of consciousness
d) mini mental status examination
e) presence of visual hallucinations
24 Sample Questions Surgery Toronto Notes 2005

Surgery
1) A 25 year-old known substance abuser is brought to the ED with a 10) Which of the following is not a sign or symptom of carpal tunnel
suspected overdose. Which of the following is not considered a syndrome :
universal antidote? a) loss of sensation to the proximal palm
a) glucose b) positive Phalen test
b) oxygen c) positive Tinel sign
c) calcium gluconate d) pain involving the thumb, index, long, and part of the ring
d) naloxone digits
e) thiamine e) delayed transit time on nerve conduction study
2) Which of the following is not a classic sign of a basal skull fracture? 11) A 35 year-old intoxicated man presents to the Emergency
a) Battle sign Department with a deep laceration to his right distal forearm after
b) racoon eyes putting his hand through a window. On examination, he has no
c) hemotympanum sensation to his little finger and the ulnar aspect of his ring finger.
d) Freedman sign Power on abduction and adduction of all fingers is markedly
e) CSF rhinnorhea/ottorrhea decreased. He is unable to flex the distal joint of his little finger.
When he flexes his wrist, his hand deviates radially. The
3) A 19 year-old female with a traumatic head injury is brought to the ED examination was otherwise normal. What structure(s) have been
by EMS. She is hemodynamically stable but requires assisted damaged?
ventilation through an endotracheal tube. She does not open her eyes a) ulnar nerve
to painful stimuli and maintains an abnormal extension posture. b) flexor carpi ulnaris and flexor digitorum profundus muscles
Which of the following is incorrect? c) median nerve
a) her GCS is 3T d) radial nerve
b) a GCS of 8 is an indication for intubation e) brachioradialis
c) isolated head injuries can cause shock
d) she should be ventilated to a pCO2 of 30-35 mm Hg 12) In which of the following are systemic antibiotics not indicated :
e) all are incorrect a) animal bite to hand
b) laceration over metacarpal-phalangeal joint sustained after
4) Conductive hearing loss is a symptom of: punching someone in a fight
a) presbycusis c) positive Finkelstein test
b) Meniere disease d) burn wound to 75% of the dorsal aspect of the hand
c) cholesteatoma e) acute suppurative tenosynovitis
d) Bell palsy
e) acoustic neuroma 13) Prolonged vomiting is associated with what electrolyte
abnormality?
5) The following statements regarding epistaxis are false EXCEPT: a) hypochloremic hypokalemic metabolic acidosis
a) Epistaxis rarely occurs in children b) hypochloremic hypokalemic metabolic alkalosis
b) It commonly results from rupture of posterior placed nasal vessels c) hyperchloremic metabolic acidosis
c) It may be treated by ligation of the ipsilateral internal carotid d) hyperkalemia
artery e) none of the above
d) Epistaxis may be treated by cautery of Little's area with silver
nitrate 14) Indications for immediate operative intervention rather than
e) It is not a common cause of emergency admission to ENT wards conservative management for arterial insufficiency include all the
following EXCEPT:
6) A 52 year-old dentist comes to your office complaining of severe a) nocturnal limb pain
bilateral buttock cramps and thigh fatigue during a tennis match and b) ischemic ulceration
recent onset of impotence The most likely diagnosis is: c) absent pulse
a) lumbosacral disc problem d) ischemic neuropathy
b) multiple sclerosis e) toe gangrene
c) Leriche syndrome
d) metastatic carcinoma of the spine 15) Surgical indications for diverticulitis include all of the following
EXCEPT:
7) A young woman who was involved in a cliff diving accident is a) peritonitis
brought into the emergency department unresponsive to deep pain b) persisting hemorrhage
with a right pupil that is dilated and nonreactive. The left pupil is c) fistula
normal. The most appropriate inital treatment is: d) greater than two severe attacks
a) endotracheal intubation e) palpable abdominal mass in left lower quadrant
b) CT scan of the head
c) 500 cc normal saline IV 16) Six days following a hemicolectomy for colorectal carcinoma, a 54
d) IV steroids year-old woman experiences calf pain unilaterally. On physical
e) IV mannitol examination, the affected side has a greater diameter than the
other, and there is tenderness to palpation. Which of the
8) Patients with an organic cause for impotence are often characterized following investigations would be the next step:
by all of the following EXCEPT: a) venogram
a) diabetes b) compression venous Doppler flow studies
b) older age c) x-ray of the affected calf
c) intermittent difficulty d) ventilation/perfusion scan
d) nocturnal penile tumescence absent e) compartment pressure monitoring
e) none of the above
17) All of the following radiologic abdominal plain film findings are
9) In the investigation of infertility, a normal semen analysis MUST consistent with mechanical bowel obstruction except:
contain: a) a "step ladder" pattern
a) a volume of 0.5-1 mL b) dilated small bowel loops
b) morphology > 85% normal forms c) air-fluid levels at uniform height in same bowel loop
c) WBC < 1 per high power field d) absence of gas in large bowel
d) motility > 75% e) rows of small gas accumulations in valvulae conniventes (i.e.
e) sperm count > 20 million sperm/mL "string of pearls")
Toronto Notes 2005 Surgery Sample Questions 25

18) After a weekend of heavy drinking, a 28 year-old male presents with 27) A patient with a subarachnoid hemorrhage (SAH) caused by a right
abdominal pain radiating to the back. What is the investigative and anterior communicating artery aneurysm undergoes successful
prognostic modality of choice for the suspected diagnosis? surgery 2 days after the hemorrhage. Three days later, right arm
a) abdominal ultrasound weakness develops. The most likely diagnosis is:
b) abdominal CT a) hydrocephalus
c) abdominal plain film b) meningitis
d) endoscopic retrograde cholangiopancreatography c) repeat hemorrhage
e) laparoscopy d) vasospasm
e) none of the above
19) An 18 year-old motorcyclist presents in the emergency department
following an accident. He has a compound tibia and fibula fracture 28) Complications of tonsillectomy include all of the following EXCEPT:
of the right leg and on examination the right leg has no pulses. Your a) secondary hemorrhage
immediate treatment should be: b) severe otalgia
a) immediate angiogram c) Quinsy
b) immediate surgery d) nasopharyngeal stenosis
c) casting and/or splinting e) none of the above
d) reduction and splinting
e) x-ray 29) Features characteristic of acute tonsilitis include all of the following
EXCEPT:
20) Which of the following is the most serious complication of a dis a) odynophagia
placed supracondylar fracture of the humerus? b) cough
a) compartment syndrome of the forearm c) referred otalgia
b) failure to heal d) cervical lymphadenopathy
c) healing in a non-anatomical position e) dysphagia
d) injury to the median nerve
e) significantly limited range of elbow motion 30) A 6 month-old baby has had mild inspiratory stridor for the last
2 months. The most likely cause of this is:
21) What is the typical audiogram finding in noise induced hearing loss: a) laryngomalacia
a) conductive loss in the low frequencies b) acute epiglottitis
b) conductive loss at 2000 Hz c) croup
c) sensorineural loss at 2000 Hz d) tonsillar hypertrophy
d) sensorineural loss at 4000 Hz e) foreign body aspiration
e) mixed broad spectrum hearing loss
31) A 48 year-old male is brought to the ED with CPR being
22) Which of the following is not a complication of untreated otitis administered. The ECG shows electrical activity present but you
media? cannot palpate a carotid pulse. Which of the following is not on the
a) cholesteatoma differential for pulseless electrical activity?
b) meningitis a) hypotension
c) tympanic membrane perforation b) hypokalemia
d) trigeminal neuralgia c) cardiac tamponade
e) conductive hearing loss d) hypothermia
e) tension pneumothorax
23) With respect to control of micturition, all of the following are true
EXCEPT: 32) A 37 year-old male arrives at the Emergency Department
a) damage to the cerebral cortex results in hyperactivity of the unconscious. He is warm and sweaty. His heart rate is 52 bpm, his
detrusor BPis 90/60. His pupils are constricted, his eyes are teary, and he is
b) the basal ganglia inhibits bladder overactivity drooling. You assume he is suffering from a toxidrome. What
c) the cerebellum coordinates emptying of the bladder antidote will you give him?
d) damage to the cerebellum results in hypotonicity of the detrusor a) Flumazenil
e) injury above the brainstem results in hyperactive bladder only b) Naloxone
c) Glucagon
24) Which of the following drugs will not promote urine retention? d) Atropine
a) sympathomimetics e) Ethanol
b) anticholinergics
c) cholinergic agonists 33) A 60 year-old man presents with back and leg pain and trouble
d) TCAs urinating. On examination, he has decreased sensation over the
e) smooth muscle depressants buttocks, normal motor power, and absent ankle jerks bilaterally.
How would this patient best be investigated?
25) A 28 year-old male presents with painless swelling in the testicle. a) plain films of the lumbar spine
On examination, it appears to be a rubbery, hard mass. He has no b) CT of lumbar spine
palpable abdominal masses. A testicular ultrasound confirms the c) MRI of spine
diagnosis of testicular tumour, and an abdominal CT reveals normal d) investigate only if no improvement after 6 weeks of symptomatic
retroperitoneal nodes. Which statement is correct with respect to this treatment
case? e) gallium scan of the spine
a) his overall prognosis is very poor
b) the testicular mass is likely a secondary tumour 34) A 34 year-old woman presents with a 3 day history of increasing
c) a needle aspiration of the tumour is indicated fever with chills, sweats, nausea, and 2 episodes of vomiting. On
d) a transcrotal approach for biopsy is contraindicated examination, she has costovertebral angle tenderness and urine
e) he will likely require chemotherapy reveals gross pyuria. Initial management should include:
a) abdominal ultrasound
26) A 78 year-old woman complains of experiencing headaches and b) abdominal CT
progressive confusion for the last month. She has a left hemianopia c) empirical treatment with antibiotics; image only if she fails to
and cannot dress herself. A CT scan demonstrates a large, irregularly respond to therapy
enhancing mass in the right parietal lobe. There is no obvious d) treat with antibiotics and perform IVP 4-6 weeks after resolution
systemic disease. The most likely diagnosis is: of illness
a) brain abscess e) immediate DTPA renal scintigraphy
b) glioblastoma multiforme
c) meningioma
d) metastasis
e) CNS lymphoma
26 Sample Questions Surgery Toronto Notes 2005

35) All of the following statements regarding knee injuries are correct 43) Which of the following is not a surgically correctable cause of
EXCEPT: hypertension?
a) locking of the knee may be due to a torn meniscus a) aortic coarctition
b) minor tears of the medial collateral ligament can be treated b) renal artery atherosclerosis
with brief immobilization then range of motion and c) pheochromocytoma
strengthening exercises d) primary hyperaldosteronism (Conn syndrome)
c) Lateral meniscus tears are more common than medial meniscal e) none of the above
tears
d) anterior cruciate ligament tears may give a positive Lachman 44) A 16 year-old female fell while roller-blading on her outstretched
test right hand. At a nearby emergency department X-rays confirmed
e) a knee dislocation may be associated with major ligament the diagnosis of a closed Colles fracture. The proper reduction
damage technique for this wrist fracture is which of the following:
a) slight extension, full pronation, and full ulnar deviation
36) An 83 year-old man has fallen while walking down stairs. He is b) slight flexion, full supination, and full radial deviation
brought to the emergency department with a 3-part c) slight extension, full supination, and full ulnar deviation
intertrochanteric hip fracture. Which of the following procedures d) slight flexion, full pronation, and full ulnar deviation
would you choose to perform? e) slight extension, full pronation, and full radial deviation
a) hemiarthoplasty
b) total hip replacement 45) Which of the following is least likely to cause avascular necrosis:
c) multiple pin fixation a) sickle cell disease
d) bipolar arthroplasty b) septic arthritis
e) pin and plate c) steroid use
d) constrictive dressings
37) A 45 year-old man with a history of polycystic kidney disease e) post-traumatic fracture
presents with painless gross hematuria. You order:
a) no investigation is required since the hematuria is most likely 46) A 24 year-old football player severely fractures his ankle while
due to the rupture of renal cyst(s) playing in a game and subsequently requires ORIF treatment.
b) no investigation at this time. Investigate if gross hematuria Indications for ORIF treatment of an ankle fracture include all of
persists the following except:
c) U/S a) a fracture-dislocation
d) U/S, urine C&S, cystoscopy b) undisplaced fracture with Grade II ATFL tear
c) trimalleolar fracture
38) A 75 year-old man with a history of nocturia has not micturated d) unstable talar tilt
for the last 10 hours, and is complaining of severe lower abdominal e) unable to maintain a closed reduction
pain. The most likely cause is:
a) BPH 47) Which of the following is not an indication for repair of an orbital
b) prostate cancer blow-out fracture?
c) renal failure a) enopthalmos
d) UTI b) persistent diplopia
e) prostatitis c) positive forced duction test
d) orbital floor lesion < 0.7 cm
39) A 43 year-old woman is brought to the Emergency Department e) other upper facial fractures
after being burned in a house fire. You estimate first degree burns
to 20% of her body, second degree burns to 11% of her body, and 48) Which of the following is true regarding mandibular fractures?
third degree burns to 9% of her body. She weighs 60 kg and is a) they are predominantly unilateral
120 cm tall. What IV therapy would you begin immediately? b) compound fractures are rarely intraoral
a) normal saline at 200 cc/hr for 24 hours c) they may manifest with numbness in the V2 facial nerve
b) normal saline at 400 cc/hr for 24 hours distribution
c) normal saline at 150 cc/hr for 8 hours, then 75 cc/hr for the d) malocclussion of teeth is a common sign
next 16 hours e) they seldom result in trismus
d) normal saline at 300 cc/hr for 8 hours, then 150 cc/hr for the
next 16 hours 49) With respect to painful scrotal swelling, all of the following are
e) normal saline at 600 cc/hr for 8 hours, then 300 cc/hr for the true EXCEPT:
next 16 hours a) torsion of testicular appendages will usually subside without
surgical intervention
40) Which of the following statements is true? b) nausea/vomiting is very common in epididymitis
a) skin grafts have their own blood supply c) ultrasound is helpful in determining the cause of hematocele
b) granulation tissue, bone, and perichondrion can all support a d) torsion most commonly occurs in young males
skin graft e) orchitis may result in testicular atrophy
c) wound contracture does not occur with skin grafting
d) axial flaps do not rely on a particular anatomically defined 50) Which is following statement about PSA is true?
vascular bundle a) every man past the age of 50 should have an annual PSA test
e) muscle flaps can aid in immunologic defense in infected as a primary screen for prostate cancer
wounds b) PSA is best used to follow disease progression or recurrence
post-operatively
41) Risk factors for hepatocellular carcinoma include all of the c) PSA is a useless test
following EXCEPT: d) increased complex PSA to total PSA ratio favors BPH over
a) hepatitis A prostate cancer
b) cirrhosis e) all of the above
c) exogenous steroid use
d) hemochromatosis 51) A 40 year-old women presents to the ER with fever, no nausea or
e) smoking vomiting, and left flank pain radiating to the groin. Body CT
reveals 8 mm stone in the left proximal ureter. You should:
42) Which of the following is associated with biliary colic? a) send her home because renal stone is benign disease
a) epigastric pain b) send her home with oral analgesics and ask her to come back if
b) rebound tenderness the pain does not resolve in the next 24 hours
c) jaundice c) admit, close monitoring, hydration and analgesics
d) Murphy sign d) admit, hydration, analgesics and IV antibiotics. (e.g. Amp and
e) all of the above Gent), and consider stenting if the symptom persists
Toronto Notes 2005 Surgery Sample Questions 27

52) Nasopharyngeal carcinoma: 60) A 24 year-old woman arrives at the Emergency Department
a) is most common in people from south-east Africa unconscious. Her BP is 90/60, her heart rate is 60 bpm, she is
b) presents early with nasal pain breathing at 8 breaths per minute and her O2 sat is 86%. Her
c) is treated by wide surgical excision eyes remain closed even after pain stimulation and the only sounds
d) is monitored by measurement of Epstein-Barr virus antibodies she makes are incomprehensible. Her elbows and wrists are flexed
e) is curable in 90% of patients with her feet extended. This patient’s GCS score is:
a) 3
53) Factors which contribute to the development of squamous cell b) 4
cancers of the head and neck include all of the following EXCEPT: c) 5
a) cigarette smoking d) 6
b) alcohol ingestion e) 7
c) aging
d) exposure to particular emission from diesel engines 61) Which physical exam finding below is usually not associated with
e) solar irradiation increasing intracranial pressure?
a) deteriorating level of consciousness
54) Which of the following statements about aspiration of a peanut b) increasing heart rate and increasing blood pressure
into the tracheobronchial tree is FALSE? c) yawning, hiccuping, vomiting
a) the peanut is more likely to be in the left lower lobe bronchus. d) seizures
b) expiratory wheeze is the most likely finding on physical exam e) unilateral sixth cranial nerve palsy
c) the peanut should be removed with the patient under general
anesthesia through an open bronchoscope with forceps 62) Which of the following is false regarding post operative wound
designed to grasp peanuts infections?
d) a chest x-ray may show atelectasis distal to the blocked a) S. aureus is the most common cause
bronchus b) usually present with fever post of day 3-4
e) pneumonia may be a complication c) increased likelihood in diabetics
d) mainly treated with antibiotics
55) Which of the following is an absolute indication for a e) risk increases with length of surgery
tonsillectomy?
a) airway obstruction 63) Which of the following signs and symptoms warrant surgical
b) recurrent (>5) episodes of tonsillitis intervention for patients with small bowel obstruction?
c) peritonsillar abscess a) abdominal tenderness
d) tonsillar hypertrophy b) air-fluid levels on abdominal x-ray
e) halitosis c) worsening abdominal pain
d) feculent vomitus
56) A 72 year-old man on physical examination is found to have e) air in the colon and rectum on abdominal x-ray
expressive dysphasia and mild right arm weakness. The most
probable location of his lesion is: 64) A 59 year-old woman presents to her family physician with a 3 cm
a) right parietal lobe palpable, well circumscribed, non-tender breast mass. She first
b) left frontal lobe noticed it several months ago and believes it has increased in size
c) right frontal lobe since then. Her mother and maternal aunt were diagnosed with
d) left parietal lobe breast cancer in their early 50's. She has no other health complaints.
e) basal ganglia A subsequent mammogram shows no abnormalities. Which of the
following is the next most appropriate step in management?
57) Inital management of any patient with coma of undetermined a) prophylactic mastectomy
cause includes all except: b) repeat mammogram in 6 months
a) clear and secure the airway c) repeat mammogram ion 1 year
b) naloxone d) excisional biopsy
c) D50W 50 mL IV e) ultrasound
d) dexamethasone 16 mg IV
e) thiamine 65) Which of the following is false regarding pancreatic pseudocysts?
a) it is caused by duct leakage
58) A 53 year-old male presents to the ED with new onset of a severe b) clinically suspected if persisting pain > 2 weeks following
headache associated with nausea and vomiting. There is no diagnosis of acute pancreatitis
history of trauma. He is alert and oriented with no neck stiffness. c) majority are treated surgically
Anisocoria is present. The most likely diagnosis is: d) surgical intervention is typically delayed to allow pseudocyst to
a) intracranial bleed mature
b) cluster headache e) lacks true epithelium
c) meningitis
d) migraine headache 66) An overweight, 45 year-old man presents with left lower quadrant
e) tension headache tenderness and a one week history of abdominal pain, loose non-
bloody stools and worsening fever. Laboratory investigation yields
59) A 16 year-old girl is brought to hospital by her frantic parents after leukocytosis with neutrophilia and left shift. What is the diagnostic
a bee sting. Vitals sings are BP 70/40, RR 30 and laboured, HR 140, modality of choice?
T 37.5. Which of the following would not be an option in her a) CT scan
management? b) barium enema
a) epinephrine c) ultrasound
b) diphenhydramine d) plain abdominal film
c) methylprednisolone e) angiography
d) salbutomol
e) atropine 67) A 19 year-old woman is brought to the emergency room following
involvement in a motor vehicle accident. On examination, she has a
GCS of 10 and swelling over the occipital protuberance. The most
appropriate imaging study is:
a) MRI of skull and contents
b) skull films
c) head CT with contrast
d) head CT without contrast
e) CT myelogram
28 Sample Questions Surgery Toronto Notes 2005

68) An 80 year-old woman suffers cervical spinal soft-tissue injury in a 75) Which of the following findings is inconsistent with cardiac
motor vehicle accident with no skeletal or neurologic damage tamponade?
documented at the time. Three months later, she presents with a) hypotension
sudden onset of homonymous right upper quadrantanopia. CT b) pulsus paradoxus
demonstrates a non-hemorrhagic lesion in the left lower occipital c) Kussmaul sign
lobe. Which imaging study would likely yield the most useful d) jugular venous distension
information? e) muffled heart sounds
a) carotid Doppler ultrasound
b) echocardiography 76) Tension pneumothorax is best diagnosed with:
c) MR angiography a) stat CT scan
d) SPECT scan b) chest x-ray
e) C-spine plain films c) watch and wait
d) clinical exam
69) Which of the following radiographic features is most consistent e) none of the above
with osteoarthritis of the knee?
a) marginal erosions 77) Which of the following is NOT a cause of major lower
b) juxta-articular osteopenia (demineralization) gastrointestinal hemorrhage?
c) loss of articular cartilage with narrowing of the radiologic joint a) diverticulitis
space b) angiodysplasia
d) osteonecrosis (avascular necrosis) of the medial femoral condyle c) aortoenteric fistula
e) syndesmophyte formation d) none of the above
70) Which of the following statements is incorrect with respect to 78) All of the following have been associated with posterior shoulder
wound healing? dislocation EXCEPT:
a) epithelialization can occur within 24 hours following primary a) ethanol
closure of a wound b) electricity
b) maximum wound strength is often achieved after 2 years c) exercise
c) wounds continue to gain strength after collagen synthesis has d) epilepsy
reached an equilibrium e) encephalitis
d) wound contracture is mediated by myofibroblasts
e) the incidence of wound infection increases with healing by 79) Management of an open fracture should always include each of the
secondary intention following EXCEPT:
a) assessment of neurovascular status
71) Which of the following is not a cause of sensorineural hearing loss: b) reduction and fixation of fracture
a) ossicular discontinuity c) irrigation and debridement of wound
b) ototoxicity d) application of sterile dressing
c) Méniére disease e) application of topical antibiotic
d) noise
e) presbycusis 80) What is the most common type of thyroid cancer?
a) medullary
72) Which of the following is not true of Meniere’s disease? b) papillary
a) characterized by quadrad of vertigo, hearing loss, tinnitus and c) follicular
aural fullness d) lymphoma
b) vertigo burns out with time e) anaplastic
c) can be treated with diuretics
d) it is the result of an abnormal buildup of potassium
e) the vertigo lasts for seconds
73) A neurosurgeon complains of a 3 week history of awakening at
night with right-hand discomfort that resolves after several minutes.
On examination, he has mild weakness of thumb abduction and
diminished pain sensibility on the palmar aspect of the thumb and
index finger. The most likely diagnosis is:
a) carpal tunnel syndrome
b) cervical radiculopathy
c) reflex sympathetic dystrophy
d) tendonitis
e) left middle cerebral artery ischemic attacks
74) A 73 year-old woman presents with a 6 month history of
deteriorating gait and low back discomfort, exacerbated by walking.
Examination is unremarkable except for hypoactive muscle stretch
reflexes in the legs. X-rays of the lumbosacral area shows the
expected degenerative changes associated with a woman of her age.
The most likely diagnosis is:
a) acute lumbar disc hernation
b) lumbar stenosis
c) myopathy
d) normal pressure hydrocephalus
e) cervical stenosis
Toronto Notes 2005 Answers to Sample Questions Sample Questions 29

Answers to Sample Questions

COMMUNITY AND PUBLIC HEALTH


1. D 14. E 27. D 40. A 53. C
2. D 15. D 28. E 41. C
3. C 16. B 29. D 42. D
4. B 17. D 30. B 43. C
5. C 18. A 31. D 44. B
6. C 19. D 32. B 45. A
7. E 20. A 33. D 46. E
8. E 21. E 34. B 47. B
9. D 22. B 35. D 48. C
10. B 23. C 36. B 49. E
11. A 24. E 37. C 50. C
12. B 25. D 38. A 51. D
13. C 26. E 39. B 52. B

MEDICINE
1. C 16. C 31. B 46. A 61. C 76. E
2. B 17. B 32. E 47. D 62. D 77. D
3. D 18. E 33. D 48. D 63. D 78. C
4. A 19. D 34. C 49. D 64. B
5. E 20. E 35. C 50. D 65. D
6. C 21. E 36. B 51 D 66. E
7. C 22. D 37. F 52. D 67. C
8. E 23. E 38. C 53. B 68. B
9. D 24. D 39. A 54. A 69. C
10. B 25. B 40. B 55. B 70. D
11. D 26. C 41. B 56. C 71. B
12. B 27. A 42. A 57. E 72. B
13. B 28. D 43. E 58. A 73. E
14. A 29. C 44. D 59. B 74. B
15. C 30. C 45. D 60. E 75. C

OBSTETRICS AND GYNECOLOGY


1. C 13. A 25. E 37. C 49. E
2. B 14. E 26. A 38. B 50. D
3. D 15. C 27. C 39. C
4. C 16. C 28. D 40. E
5. D 17. D 29. E 41. B
6. C 18. B 30. C 42. D
7. E 19. B 31. B 43. B
8. C 20. C 32. D 44. B
9. E 21. E 33. D 45. B
10. B 22. C 34. D 46. D
11. E 23. B 35. C 47. B
12. B 24. E 36. A 48. A
30 Sample Questions Answers to Sample Questions Toronto Notes 2005

PEDIATRICS
1. E 17. B 33. C 49. E 65. B 81. B
2. A 18. B 34. C 50. D 66. E 82. D
3. A 19. B 35. D 51. A 67. C 83. E
4. C 20. B 36. A 52. B 68. E
5. A 21. A 37. C 53. D 69. B
6. C 22. A 38. C 54. E 70. A
7. B 23. E 39. A 55. A 71. B
8. E 24. B 40. C 56. E 72. D
9. D 25. D 41. E 57. D 73. A
10. A 26. C 42. A 58. C 74. A
11. B 27. B 43. B 59. C 75. B
12. E 28. C 44. D 60. C 76. B
13. D 29. A 45. C 61. D 77. C
14. B 30. B 46. A 62. E 78. B
15. A 31. B 47. B 63. A 79. D
16. B 32. E 48. B 64. E 80. D

PSYCHIATRY
1. D 15. E 29. A 43. D
2. A 16. B 30. A 44. E
3. D 17. D 31. B 45. D
4. C 18. A 32. C 46. B
5. E 19. D 33. E 47. D
6. E 20. E 34. D 48. C
7. B 21. D 35. E 49. C
8. B 22. D 36. E 50. B
9. B 23. C 37. D 51. D
10. D 24. C 38. B 52. D
11. B 25. E 39. D 53. C
12. E 26. E 40. D 54. C
13. D 27. C 41. A 55. A
14. C 28. E 42. E

SURGERY
1. C 15. E 29. B 43. D 57. D 71. A
2. D 16. B 30. A 44. D 58. A 72. E
3. C 17. C 31. B 45. B 59. E 73. A
4. C 18. B 32. D 46. D 60. D 74. B
5. D 19. D 33. C 47. D 61. B 75. C
6. C 20. A 34. C 48. D 62. D 76. D
7. A 21. D 35. C 49. B 63. C 77. A
8. C 22. D 36. E 50. B 64. D 78. C
9. E 23. D 37. D 51. D 65. C 79. E
10 A 24. C 38. A 52. D 66. A 80. B
11. A 25. D 39. D 53. D 67. D
12. D 26. B 40. E 54. A. 68. C
13. B 27. D 41. A 55. A 69. C
14. C 28. C 42. A 56. B 70. E
Toronto Notes 2005 Key Feature Questions Sample Questions 31

Key Feature Questions


Instructions for Examinees
“Key Feature” questions can pertain to solutions of clinical cases that involve data-gathering (e.g., history taking, physical examination, laboratory
investigations), diagnosis or treatment.
The Key Feature questions will ask for your answers in different ways: you may be asked to (i) select the correct response(s) from a menu of options, or
(ii) print the correct answer(s) on a blank line.
Please note the following guidelines before proceeding:
1. After reading the description of the case, read the question carefully. Take note as to whether you are told the maximum number of answers to
provide. If you exceed this number, you will receive no credit for the question. If you select or list fewer answers, you will still receive credit
for any of your answers which are correct.
2. If you are asked to select your answers from a menu:
a. Read the complete menu of options before recording your answers.
b. Make certain that the number of answers you record is not greater than the number you have been asked to select.
3. If you are asked to list your answers below the question:
a. Ensure that you print each answer legibly.
b. Record each answer on a separate blank line.
c. Do not record more than the maximum number you are asked to list.

CASE1
A 30 year-old homeless male presents with a 4 week history of feeling
generally unwell with anorexia, weight loss and intermittent fever. He
lives primarily on the streets or occasionally in a men’s shelter. He
admits to regular use of intravenous heroin for the last seven years and
often shares needles.

QUESTION 1 (CASE 1) QUESTION 2 (CASE 1)


Given this history, which of the following diagnoses would you consider? A mitral regurgitant murmur is auscultated in this patient.
Select up to five. His temperature is 39.0ºC.
1. Acute pancreatitis Which of the following other findings may be found on physical
2. AIDS related complex examination given the diagnoses being considered?
3. Aspergillosis Select up to five.
4. Bacterial endocarditis 1. Ascites
5. Chronic pancreatitis 2. Campbell De Morgan’s spots
6. Delirium tremens 3. Cannon “a” waves
7. Diabetes mellitus 4. Cyanosis
8. Hepatitis B 5. Erythema nodosum
9. Hepatocellular carcinoma 6. Follicular keratosis
10. Histoplasmosis 7. Gynecomastia
11. Leukemia 8. Hepatomegaly
12. Lymphoma 9. Janeway lesions
13. Osteomyelitis 10. Jaundice
14. Peptic ulcer disease 11. Koilonychia
15. Pericarditis 12. Livedo reticularis
16. Pyelonephritis 13. Osler nodes
17. Renal failure 14. Pallor
18. Secondary syphilis 15. Palmar erythema
19. Talc pneumonitis 16. Papilledema
20. Tuberculosis 17. Purpura
18. Roth spots
19. Spider nevi
20. Splenomegaly
32 Sample Questions Key Feature Questions Toronto Notes 2005

CASE 2 CASE 3
A babysitter brings a 14 month-old to emergency because of incon- An 84 year-old female is brought to the Emergency Department by her
solable crying. The child appears well cared for and healthy but a daughter who states that her mother has not been herself over the past
"bruise" is noted on the left humerus and a well-circumscribed small three days. Further questioning reveals that the mother has been less
fresh burn is noted on the other shoulder. attentive than usual and more withdrawn, her conversation is often
rambling and she has not been sleeping well. Prior to this, she func-
tioned very well.
QUESTION 3 (CASE 2)
QUESTION 6 (CASE 3)
List four well recognized risk factors for child abuse:
What diagnoses would you consider at this time? List up to two.
1. _____________________________________________________________
1. _____________________________________________________________
2. _____________________________________________________________
2. _____________________________________________________________
3. _____________________________________________________________
4. _____________________________________________________________
QUESTION 7 (CASE 3)
What additional aspects of history would be most important to know?
QUESTION 4 (CASE 2) Select up to four.
1. Alcohol use
Which of these physical findings are most suggestive of child abuse? 2. Complete psychiatric history
Select up to four. 3. Family history of Alzheimer disease
1. Alopecia 4. Financial status
2. Blue sclera 5. History of fever
3. Bruises on shins and elbows 6. History of incontinence
4. Buttocks wasting 7. History of osteoarthritis
5. Caput medusa 8. History of Parkinson disease
6. Clubbing 9. History of previous CVA
7. Condylomata acuminata 10. Immunization status
8. Eczema 11. Marital status
9. Full fontanelle 12. Medication history
10. Healed laceration on chin 13. Occupational history
11. Hutchison teeth 14. Pattern of symptoms over the course of the day, e.g. fluctuations
12. Impetigo 15. Presence of hallucinations
13. Large purple macular lesion on buttock 16. Previous history of depression
14. Limp 17. Previous seizure disorder
15. Oral thrush 18. Previous surgery
16. Petechiae 19. Recent travel
17. Pitted finger nails 20. Smoking history
18. Proptosis
19. Retinal hemorrhage
20. Seborrhea
QUESTION 8 (CASE 3)
The interview confirms the daughter's description of the mother.
QUESTION 5 (CASE 2) Physical exam reveals: BP 120/80 supine and 100/70 sitting, RR=22,
P=104, T=38.7ºC. JVP is at the sternal angle. Respiratory exam reveals
You suspect this is a case of child abuse. Which of the following are dullness, increased tactile fremitus, crackles and bronchial breath
appropriate steps in the initial management. Select up to four. sounds all in the left base. Heart sounds are normal. The abdominal
1. Abdominal ultrasound exam is normal. CNS exam does not reveal any focal findings. What
2. Ascorbic acid level would you include in your initial investigations? Select up to four.
3. Bone density studies 1. ALT
4. CBC 2. AST
5. Chest x-ray 3. Blood cultures
6. Coagulation studies 4. B12
7. Creatinine 5. CBC
8. Dietary history 6. Chest x-ray
9. ECG 7. CPK
10. Factor 8 assay 8. CT scan of head
11. Liver spleen scan 9. Drug screen
12. MRI 10. EEG
13. Notify CAS (Children’s Aid Society) 11. Electrolytes
14. Notify police 12. HIV test
15. Notify public health nurse 13. MRI of head
16. Ophthalmology consult 14. Protein electrophoresis
17. Psychiatry consult 15. PT and PTT
18. Serum calcium, phosphorus and alkaline phosphorus 16. RBC folate
19. Skeletal survey 17. SPECT scan
20. Urinalysis 18. TSH
19. Urea and creatinine
20. VDRL
Toronto Notes 2005 Key Feature Questions Sample Questions 33

CASE 4 CASE 5
A 57 year-old man presents to the Emergency Department with a 12 A 30 year-old woman comes to your office with a 3 day history of vul-
hour history of left flank pain. var and vaginal pruritus.

QUESTION 9 (CASE 4) QUESTION 12 (CASE 5)


What diagnoses would you consider at this time? What diagnosis would you consider at this time? List up to three.
Select up to four.
1. Ruptured abdominal aortic aneurysm 1. _____________________________________________________________
2. Bowel obstruction
3. Appendicitis 2. _____________________________________________________________
4. Pyelonephritis
5. Diverticulitis 3. _____________________________________________________________
6. Renal calculi
7. Biliary colic
8. Renal cell carcinoma
9. Pneumonia QUESTION 13 (CASE 5)
What features on the history would you inquire about to make a diag-
nosis? Choose up to seven.
QUESTION 10 (CASE 4) 1. Date of last menstrual period
2. Sexual history
On history and physical exam, what symptoms and signs might aid in 3. Vaginal discharge
diagnosis? Choose up to seven. 4. Vulvar erythema
1. Fever/chills 5. Vaginal odor
2. Weight loss 6. Recent antibiotic therapy
3. Rebound tenderness 7. Diabetes
4. Costovertebral angle tenderness 8. Macular rash
5. Urgency and frequency 9. Psychiatric history
6. Pruritus 10. Headache
7. Hypotension 11. Allergies
8. Pulsatile abdominal mass 12. Dysuria
9 Nausea and vomiting 13. Use of douches
10. Left flank mass 14. Sore throat
11. Productive cough 15. Previous history of STDs
12. Abdominal guarding
13. Pale stools and dark urine
14. Hematuria
QUESTION 14 (CASE 5)
What tests would you do to confirm the diagnosis? Choose up to four.
QUESTION 11 (CASE 4) 1. Urine C&S
2. Wet prep of vaginal discharge for microscopy
What tests might be necessary to confirm your diagnosis? 3. Urinalysis
Choose up to six. 4. CBC
1. ESR 5. ESR
2. Serum amylase 6. Culture for gonorrhea
3. Urine R&M 7. Culture for chlamydia
4. Chest x-ray 8. Abdominal CT
5. Abdominal/pelvic ultrasound 9. Pelvic ultrasound
6. 3 views of the abdomen 10. Culture for Trichomonas vaginalis
7. Urine C&S
8. Abdominal CT
9. IVP
10. Total bilirubin
11. Electrolytes
12. 24 hour creatinine clearance
13. KUB
14. Serum Mg
15. Serum alkaline phosphatase
16. Barium enema
17. Serum Ca
18. CBC
34 Sample Questions Key Feature Questions Toronto Notes 2005

CASE 6 QUESTION 18 (CASE 7)


A 65 year-old man in the emergency department has RUQ pain. He is What questions would you ask on history to help determine the cause
otherwise healthy. of the bleeding? Choose up to five.
1. Family history of inflammatory bowel disease
2. Hematemesis
3. Recent weight loss
QUESTION 15 (CASE 6) 4. Nausea and vomiting
5. Change in bowel habits
List three diagnoses you should consider in this patient. 6. Abdominal pain
7. Fever/chills
1. _____________________________________________________________ 8. Family history of colo-rectal cancer
9. Blood mixed with stool or coating the surface
2. _____________________________________________________________ 10. Past history of hemorrhoids
11. Dietary history
3. _____________________________________________________________ 12. Smoking history
13. Pain on defecation
14. Allergies

QUESTION 16 (CASE 6)
What are the most important questions to ask about on history?
Choose up to seven.
1. Pruritus
2. Pale stools CASE 8
3. Fever/chills
4. Hematuria You are called to the nursery to assess a 2 hour old term newborn with
5. Jaundice respiratory distress. The only information the nurse gives you over the
6. Anorexia phone is that the baby was born this morning, had good Apgars and
7. Dark urine arrived in the nursery pink on room air and in no distress.
8. Weight loss
9. Back pain
10. Diarrhea QUESTION 19 (CASE 8)
11. Alcohol/drug history
12. Nausea and vomiting What are the most common diagnoses that would be in your differen-
13. Fatigue tial for this infant? List up to three.
14. Cold intolerance
1. _____________________________________________________________
2. _____________________________________________________________
3. _____________________________________________________________

CASE 7
A 60 year-old woman comes to your office because she has seen bright QUESTION 20 (CASE 8)
red blood in the toilet bowl after bowel movements for the past month.
She is otherwise healthy. With respect to the diagnosis, which elements of the history and physi-
cal will be most important in determining the etiology of this baby’s
distress? Select up to four.
1. Delivery mode (C-section, vaginal)
QUESTION 17 (CASE 7) 2. Feeding pattern
3. Grunting
What are three common diagnoses you should consider? 4. Head circumference
5. Heart sounds
1. _____________________________________________________________ 6. History of maternal diabetes
7. Indrawing
2. _____________________________________________________________ 8. Maternal temperature
9. Meconium stained amniotic fluid
3. _____________________________________________________________ 10. Oxygen requirement
11. Presence of bruising
12. Presenting part (breech, vertex)
13. Previous obstetric history
14. Tachypnea
15. Vital signs
Toronto Notes 2005 Key Feature Questions Sample Questions 35

CASE 9 QUESTION 22 (CASE 9)


A 45 year-old migrant farm worker comes to see you with sudden What elements of the physical exam would you focus on?
onset of severe pain in his back and right leg after lifting some bales of Choose up to four.
hay. The pain radiates into the lateral aspect of the right foot and is 1. Examination of skin
associated with numbness along the lateral aspect of the foot. He has 2. Range of motion of lumbar spine
had a 5 year history of lower back pain with a similar episode of pain 1 3. Blood pressure
year ago which was concentrated at the posterolateral aspect of the 4. Inspection of muscle bulk in lower extremities
right calf and lateral aspect of the right foot. It became progressively 5. Sensory exam of arms
worse, then resolved over a 3 week period. 6. Peripheral pulses
7. Reflexes in lower extremities
8. Motor testing of legs
9. Motor testing of arms
QUESTION 21 (CASE 9) 10. Sensory exam of legs
11. Bowstring sign
What elements of the history would you ask about in order to formu- 12. Respiratory rate
late your diagnosis? Choose up to five. 13. Temperature
1. Diffuse muscle cramps 14. Straight leg raise test
2. Alcohol intake 15. Abdominal exam
3. Effect of NSAIDs 16. Rectal exam
4. Family history of back problems 17. Pulse
5. Previous history of back injury 18. Range of motion of knee
6. Shoulder pain 19. Auscultation of chest
7. Weakness in right leg 20. Romberg test
8. Allergies
9. Knee problems
10. Urinary incontinence
11. Skin rashes
12. Previous effective physiotherapy
13. Headaches
14. Nocturia
36 Sample Questions Key Feature Scoring Guide Toronto Notes 2005

Key Feature Scoring Guide


QUESTION 1, CASE 1 QUESTION 5, CASE 2
1. Maximum Number to be selected: = 5 1. Maximum Number to be selected: = 4

2. The Number you selected (S): = ___ 2. The Number you selected (S): = ___

3. Is “S” greater than the Maximum? YES Your Score = 0. 3. Is “S” greater than the Maximum? YES Your Score = 0.
NO Continue. NO Continue.

4. There are 5 correct answers: 2, 4, 8, 13, 20 4. There are 4 correct answers: 6, 13, 16, 19

5. Number of correct answers you selected: = ___ 5. Number of correct answers you selected: = ___

QUESTION 2, CASE 1 QUESTION 6, CASE 3


1. Maximum Number to be selected: = 5 1. Maximum Number to be listed: = 2

2. The Number you selected (S): = ___ 2. The Number you listed (L): = ___

3. Is “S” greater than the Maximum? YES Your Score = 0. 3. Is “L” greater than the Maximum? YES Your Score = 0.
NO Continue. NO Continue.

4. There are 5 correct answers: 9, 13, 14, 18, 20 4. There are 2 correct answers:

5. Number of correct answers you selected: = ___ 1. Delirium


2. Acute Confusional State
5. Number of correct answers you listed: = ___
QUESTION 3, CASE 2
1. Maximum Number to be listed: = 4
QUESTION 7, CASE 3
2. The Number you listed (L): = ___
1. Maximum Number to be selected: = 4
3. Is “L” greater than the Maximum? YES Your Score = 0.
NO Continue. 2. The Number you selected (S): = ___

4. There are 8 correct answers: 3. Is “S” greater than the Maximum? YES Your Score = 0.
NO Continue.
1. Prematurity
2. Difficult child 4. There are 5 correct answers: 1, 5, 12, 14, 15
3. History of parental child abuse
4. History of substance abuse 5. Number of correct answers you selected: = ___
5. Social isolation
6. Developmental delay
7. Poverty
8. Parental character/personality disorder QUESTION 8, CASE 3
5. Number of correct answers you listed: = ___ 1. Maximum Number to be selected: = 4
2. The Number you selected (S): = ___

QUESTION 4, CASE 2 3. Is “S” greater than the Maximum? YES Your Score = 0.
NO Continue.
1. Maximum Number to be selected: = 4
4. There are 5 correct answers: 3, 5, 6, 11, 19
2. The Number you selected (S): = ___
5. Number of correct answers you selected: = ___
3. Is “S” greater than the Maximum? YES Your Score = 0.
NO Continue.
4. There are 4 correct answers: 7, 9, 14, 19
5. Number of correct answers you selected: = ___
Toronto Notes 2005 Key Feature Scoring Guide Sample Questions 37

QUESTION 9, CASE 4 QUESTION 13, CASE 5


1. Maximum Number to be selected: = 4 1. Maximum Number to be selected: = 7
2. The Number you selected (S): = ___ 2. The Number you selected (S): = ___
3. Is “S” greater than the Maximum? YES Your Score = 0. 3. Is “S” greater than the Maximum? YES Your Score = 0.
NO Continue. NO Continue.
4. There are 4 correct answers: 1, 4, 6, 8 4. There are 8 correct answers: 2, 3, 5, 6, 7, 12,
13, 15
5. Number of correct answers you selected: = ___
5. Number of correct answers you selected: = ___

QUESTION 10, CASE 4 QUESTION 14, CASE 5


1. Maximum Number to be selected: = 7 1. Maximum Number to be selected: = 4
2. The Number you selected (S): = ___ 2. The Number you selected (S): = ___
3. Is “S” greater than the Maximum? YES Your Score = 0. 3. Is “S” greater than the Maximum? YES Your Score = 0.
NO Continue. NO Continue.
4. There are 8 correct answers: 1, 2, 4, 5, 7, 8, 4. There are 4 correct answers: 2, 6, 7, 10
10, 14
5. Number of correct answers you listed: = ___
5. Number of correct answers you selected: = ___

QUESTION 15, CASE 6


QUESTION 11, CASE 4 1. Maximum Number to be listed: = 3
1. Maximum Number to be selected: = 6 2. The Number you listed (L): = ___
2. The Number you selected (S): = ___ 3. Is “L” greater than the Maximum? YES Your Score = 0.
NO Continue.
3. Is “S” greater than the Maximum? YES Your Score = 0.
NO Continue. 4. There are 3 correct answers:
4. There are 7 correct answers: 3, 5, 7, 8, 9, 13, 18 1. Biliary colic
2. Acute cholecystitis
5. Number of correct answers you selected: = ___ 3. Bile duct carcinoma
5. Number of correct answers you listed: = ___

QUESTION 12, CASE 5


1. Maximum number to be selected = 3 QUESTION 16, CASE 6

2. The number you listed (L): = ___ 1. Maximum Number to be selected: = 7


2. The Number you selected (S): = ___
3. Is “L” greater than the Maximum? YES Your Score = 0.
NO Continue. 3. Is “S” greater than the Maximum? YES Your Score = 0.
NO Continue.
4. There are 4 possible answers:
4. There are 7 correct answers: 1, 2, 3, 5, 6, 7, 8
1. Bacterial vaginosis
2. Vaginal/vulvar candidiasis 5. Number of correct answers you selected: = ___
3. Trichomonas vaginalis
4. Chemical vaginitis
5. Number of correct answers you listed: = ___
38 Sample Questions Key Feature Scoring Guide Toronto Notes 2005

QUESTION 17, CASE 7 QUESTION 20, CASE 8


1. Maximum Number to be listed: = 3 1. Maximum Number to be selected: = 4
2. The Number you listed (L): = ___ 2. The Number you selected (S): = ___
3. Is “L” greater than the Maximum? YES Your Score = 0. 3. Is “S” greater than the Maximum? YES Your Score = 0.
NO Continue. NO Continue.
4. There are 5 correct answers: 4. There are 4 correct answers: 1, 6, 8, 9
1. Hemorrhoids 5. Number of correct answers you selected: = ___
2. Anal fissure
3. Colon or rectal cancer
4. Proctitis or IBD or colitis
5. Diverticulosis QUESTION 21, CASE 9
5. Number of correct answers you listed: = ___ 1. Maximum Number to be selected: = 5
2. The Number you selected (S): = ___
QUESTION 18, CASE 7 3. Is “S” greater than the Maximum? YES Your Score = 0.
NO Continue.
1. Maximum Number to be selected: = 5
4. Essential answers are: 7 and 10
2. The Number you selected (S): = ___
5. Did you select both essential answers?
3. Is “S” greater than the Maximum? YES Your Score = 0. NO Your Score = 0.
NO Continue. YES Continue.
4. There are 6 correct answers: 1, 5, 8, 9, 10, 13 6. There are 5 correct answers: 3, 5, 7, 10, 12
5. Number of correct answers you selected: = ___ 7. Number of correct answers you listed: = ___

QUESTION 19, CASE 8 QUESTION 22, CASE 9


1. Maximum Number to be listed: = 3 1. Maximum Number to be selected: = 6
2. The Number you listed (L): = ___ 2. The Number you selected (S): = ___
3. Is “L” greater than the Maximum? YES Your Score = 0. 3. Is “S” greater than the Maximum? YES Your Score = 0.
NO Continue. NO Continue.
4. There are 4 correct answers: 4. There are 8 correct answers: 2, 4, 7, 8, 10, 11,
14, 16
1. Transient tachypnea of the newborn or wet lung
2. Meconium aspiration 5. Number of correct answers you selected: = ___
3. Pneumonia
4. Cardiac disease
5. Number of correct answers you listed: = ___

Das könnte Ihnen auch gefallen